Docsity
Docsity

Prepare for your exams
Prepare for your exams

Study with the several resources on Docsity


Earn points to download
Earn points to download

Earn points by helping other students or get them with a premium plan


Guidelines and tips
Guidelines and tips

SAEM Exam Questions 2024-2025, Exams of Nursing

A collection of sample exam questions and answers for the society for academic emergency medicine (saem) exam in 2024-2025. The questions cover a wide range of emergency medicine topics, including differential diagnoses, diagnostic tests, treatment options, and management of various medical conditions. Detailed explanations and rationales for the correct answers, making it a valuable resource for medical students, residents, and emergency medicine practitioners preparing for the saem exam. The comprehensive nature of the content, the focus on real exam questions, and the verified answers make this document potentially useful as study notes, lecture notes, summaries, or even as a cheat sheet or assignment for those preparing for the saem exam.

Typology: Exams

2024/2025

Available from 10/21/2024

Docgiana
Docgiana 🇺🇸

3.3

(12)

681 documents

1 / 425

Toggle sidebar

Related documents


Partial preview of the text

Download SAEM Exam Questions 2024-2025 and more Exams Nursing in PDF only on Docsity! SAEM EXAM QUESTIONS 2024-2025 ACTUAL EXAM REAL EXAM 1040 QUESTIONS AND CORRECT DETAILED ANSWERS WITH RATIONALES (VERIFIED ANSWERS) "Regarding the diagnosis of acute appendicitis, all the following are true EXCEPT: A. Vital signs are usually abnormal, even early in the course of acute appendicitis. B. Rebound is usually elicited only after the appendix has ruptured or infarcted. C. Rovsing's sign is pain in the right lower quadrant upon palpation of the left lower quadrant. D. The obturator sign is pain upon flexion and internal rotation of the hip. E. The psoas sign is pain upon extension of the hip." - correct answer "A. Vital signs are usually abnormal, even early in the course of acute appendicitis. The answer is A. The presentation of acute appendicitis varies tremendously. Early in its course, vital signs including temperature may be normal. Once perforation has occurred, the rate of low-grade fever (<38 C) increases to about 40%. Other variations in presentation include pain in the right upper quadrant, typically from a retrocecal or retroiliac appendix.";"Rosving's sign is described as: A. Tenderness in the right upper quadrant that is worse with inspiration. B. Pelvic pain upon flexion of the thigh while the patient is supine. C. Pelvic pain upon internal and external rotation of the thigh with the knee flexed. D. Pain that increases with the release of pressure of palpation. E. Pain in the right lower quadrant when left lower quadrant is palpated." - correct answer "E. Pain in the right lower quadrant when left lower quadrant is palpated. The answer is E. Rosving's sign is pain in the right lower quadrant when the left lower quadrant is palpated. Rebound tenderness occurs with the release of pressure. The iliopsoas sign is pain associated with thigh flexion. The obturator sign is pain that occurs with thigh rotation. All of these signs are associated with appendicitis. Murphy's sign is cessation of inspiration during palpation of the right upper quadrant and is associated with acute cholecystitis.";"In establishing a differential diagnosis of abdominal pain, which of the following is true? A. Radiation of pain to the scapula is suggestive of acute hepatitis. B. Cervical motion tenderness is a useful physical finding for differentiating women with or without acute appendicitis. SAEM EXAM QUESTIONS 2024-2025 ACTUAL EXAM REAL EXAM 1040 QUESTIONS AND CORRECT DETAILED ANSWERS WITH RATIONALES (VERIFIED ANSWERS) C. In patients with sickle cell anemia who present with abdominal pain and diarrhea, shigellosis should be a top consideration. D. The onset of pain prior to the occurrence of nausea and vomiting is more often suggestive of a surgical etiology. E. Diverticulitis tends to cause pain in the right upper quadrant." - correct answer "D. The onset of pain prior to the occurrence of nausea and vomiting is more often suggestive of a surgical etiology. The answer is D. Pain prior to nausea and vomiting is often suggestive of a surgical etiology of the pain, such as small bowel obstruction. Cervical motion tenderness has been noted in up to 25% of women with acute appendicitis. Patients with sickle cell anemia are prone to Salmonella infections. Radiation of pain to the scapula is classically present in acute choleycystitis. Diverticulitis pain is generally located in the left lower quadrant.";"Of the following pain patterns, which is the least likely associated with diagnosis of peptic ulcer disease? A. non-radiating, burning epigastric pain B. pain that awakens a patient in the middle of the night C. unrelenting pain over a period of weeks D. relief of abdominal pain with antacids E. pain that is worse preceding a meal" - correct answer "C. unrelenting pain over a period of weeks The answer is C. Pain from peptic ulcer disease typically occurs in periods of exacerbation and remission. Unrelenting pain over weeks or months should suggest an alternative diagnosis. Pain is classically described as non-radiating, burning epigastric pain. Some patients may also complain of chest or back pain. Pain is frequently severe enough to awaken patients from sleep in early morning hours but is often not present upon waking in the morning, as gastric acid secretion peaks around 2 a.m. and nadirs upon awakening.";"A 78 year old female presents to the E.D. with a sensation of left-lower quadrant abdominal pain, accompanied by some irregular bowel movements and loss of appetite. Her abdominal CT (two images) is shown in the Figure. What is the most likely diagnosis? A. ovarian cyst SAEM EXAM QUESTIONS 2024-2025 ACTUAL EXAM REAL EXAM 1040 QUESTIONS AND CORRECT DETAILED ANSWERS WITH RATIONALES (VERIFIED ANSWERS) B. Small bowel involvement is rare C. Bleeding is common due to superficial bowel wall inflammation D. There is a small increased risk of colon cancer" - correct answer "D. There is a small increased risk of colon cancer The answer is D. Although Crohn's disease may involve the entire bowel tract, the rectum is rarely involved. Involved areas are typically non-contiguous (known as "skip lesions") and the inflammation involves all of the layers of the bowel wall--resulting in many of the complications of Crohn's such as abscess and fistula formation, intestinal obstruction, and perforation. The risk of colon cancer is only slightly elevated above baseline. In contrast, Ulcerative colitis begins in the rectum and may spread to the upper parts of the colon but never involves the small intestine. The ulcerations are contiguous and involve only the colonic mucosa. The incidence of colon cancer may be increased up to 30 times over baseline.";"A 53 year old obese woman presents to the emergency department, accompanied by three of her children, complaining of severe abdominal pain that began this afternoon after lunch. Physical exam reveals marked RUQ tenderness. Likely findings on this patient would include all of the following EXCEPT: A. positive sonographic Murphy's sign B. pain in the right scapula C. leukocytosis with left shift D. marked inguinal lymphadenopathy E. aminotransferases and bilirubin within normal limits" - correct answer "D. marked inguinal lymphadenopathy The answer is D. This woman is likely suffering from acute cholecystitis. Predisposing factors include female gender, obesity, increased age and increased parity. Inflammation of the gallbladder causes RUQ pain and sonographic Murphy's sign (inspiratory arrest, due to pain, while the ultrasound probe is positioned over the gallbladder). Pain may radiate to the right scapula. Lab studies usually show leukocytosis with or without a left shift, and aminotransferases and bilirubin are usually within normal limits.";"A 25 year old female presents with epigastric pain radiating straight through to the back. Laboratory tests are notable only for markedly elevated amylase and lipase. An abdominal X-ray is taken (see Figure). Regarding this patient's presentation, which of the following is most likely true? SAEM EXAM QUESTIONS 2024-2025 ACTUAL EXAM REAL EXAM 1040 QUESTIONS AND CORRECT DETAILED ANSWERS WITH RATIONALES (VERIFIED ANSWERS) [image] A. The most likely explanation for her symptoms is gallstone-related pancreatitis. B. She probably has an ulcer, since the laboratory results are nonspecific. C. Alcohol use is only associated with pancreatitis in patients older than this woman, and who have been abusing alcohol for years. D. The abdominal X-ray is concerning for early bowel obstruction. E. The X-ray reveals that intrathoracic pathology is likely the cause of the patient's symptoms." - correct answer "A. The most likely explanation for her symptoms is gallstone-related pancreatitis. The answer is A. The X-ray reveals stones in the gallbladder. These particular stones are not likely the cause of pancreatitis, but the demonstration of gallstone disease raises the likelihood that the patient's pancreatitis is indeed due to gallstones. In the U.S., the most common etiologies of pancreatitis include gallstones (45%) and alcoholism (35%). Alcoholic pancreatitis may occur in young patients as well as in older abusers of alcohol. Many other drugs, infectious agents, and conditions are associated with the development of pancreatitis. A few examples include hypertriglyceridemia, trauma, pregnancy, pancreatic carcinoma, atherosclerotic emboli, and scorpion bites.";"A 45 year old woman presents with right upper quadrant pain and fever. The pain is worse after eating. On physical exam she has a Murphy's sign. The most likely diagnosis is: A. Appendicitis B. Diverticulitis C. Cholelithiasis D. Cholecystitis E. Mesenteric Ischemia" - correct answer "D. Cholecystitis The answer is D. Right upper quadrant pain, fever and a Murphy's sign suggests cholecystitis. Cholelithiasis presents with similar pain, but is not associated with fever or a Murphy's sign";"A 47 year-old male presents, confused, to the ED. He has limited ability to give a history. On physical examination of the skin, it is noted that there are erythematous changes to both palms. Also, the face and arms are characterized by a number of superficial, tortuous arterioles SAEM EXAM QUESTIONS 2024-2025 ACTUAL EXAM REAL EXAM 1040 QUESTIONS AND CORRECT DETAILED ANSWERS WITH RATIONALES (VERIFIED ANSWERS) which fill from the center outwards. The examination of the abdomen reveals violaceous lines radiating from the umbilicus, and there are generally increased venous markings on the abdominal wall (see Figure). What is the most likely diagnosis? [image] A. Rocky Mountain spotted fever B. necrotizing fasciitis C. liver disease D. lymphangitis E. hyperthermia" - correct answer "C. liver disease The answer is C. The patient's palmar erythema, spider angiomata, and caput medusa (due to recanalization of the umbilical vein) are all characteristic of hepatic disease. The figure demonstrates abdominal wall venous engorgement, as well as ascites (another clue to the patient's liver disease).";"A 57-year-old homeless woman with a history of schizophrenia presents to the emergency department complaining of nausea and severe abdominal pain for 48 hours. The patient is not cooperative with an upright abdominal image, so a flat plate (as shown in the Figure) is obtained. Which of the following is the most likely operative finding in this patient? [image] A. Inflamed appendix B. Rectus sheath hematoma C. Ruptured spleen D. Small bowel obstruction" - correct answer "D. Small bowel obstruction The answer is D. Dilated loops of small bowel with air-fluid levels (which are not well-seen on a flat plate) indicate small bowel obstruction. KUB is not often useful in the diagnosis of appendicitis, ruptured spleen, gallstone disease, or a rectus sheath hematoma (which is an abdominal wall condition most likely seen in anticoagulated patients with trauma or coughing). Despite this woman's history of schizophrenia and possibly diminished ability to relate a clear story of her pain, her complaint of abdominal pain must be taken seriously with a high suspicion for underlying pathology.";"All of the following factors predispose to cecal volvulus EXCEPT: SAEM EXAM QUESTIONS 2024-2025 ACTUAL EXAM REAL EXAM 1040 QUESTIONS AND CORRECT DETAILED ANSWERS WITH RATIONALES (VERIFIED ANSWERS) with conservative treatment, the elderly are at higher risk of perforation and should be admitted. Guaiac positive stool in seen in up to 50% of patients with diverticulitis. There is no reason to suspect acute blood loss requiring transfusion in diverticulitis.";"Regarding esophageal perforation, which of the following is INCORRECT: A. Esophageal perforation has been reported as a complication of nasogastric tube placement, endotracheal intubation, and esophagotracheal Combitube intubation. B. Esophageal perforation may result from forceful vomiting, coughing, childbirth or heavy lifting. C. Over 80% of esophageal perforations are iatrogenic, usually as complications of upper endoscopy, dilation, or sclerotherapy. D. Over 90% of spontaneous esophageal perforations occur in the proximal esophagus. E. Iatrogenic perforations of the esophagus usually occur in the proximal esophagus or esophagogastric junction." - correct answer "D. Over 90% of spontaneous esophageal perforations occur in the proximal esophagus. The answer is D. Over 90% of spontaneous esophageal perforations occur in the distal esophagus, whereas iatrogenic perforations are frequently at the pharyngoesophageal junction or the esophagogastric junction. Foreign body or caustic substance ingestion, severe blunt injury or penetrating trauma, and carcinoma are other causes of esophageal perforation.";"Working in the ED, you have identified a bony object wedged in the mid-esophagus of a 45 year old patient. Failure to promptly remove a foreign body impacted in the esophagus could result in: A. Esophageal perforation and mediastinitis B. Epiglottal edema and airway obstruction C. The rapid development of xerostomia D. Barrett's esophagitis" - correct answer "A. Esophageal perforation and mediastinitis The answer is A. The complications of esophageal foreign bodies are rare but serious. They include esophageal erosion and perforation, mediastinitis, esophagus-to-trachea or esophagus- to-vasculature fistula formation, stricture formation, diverticuli formation, and tracheal compression (from both the esophageal foreign body and resultant edema or infection). Air SAEM EXAM QUESTIONS 2024-2025 ACTUAL EXAM REAL EXAM 1040 QUESTIONS AND CORRECT DETAILED ANSWERS WITH RATIONALES (VERIFIED ANSWERS) trapping is a sign of a foreign body of the airway. Rarely, airway foreign bodies act as one-way valves that could cause hyperinflation of a lung segment, with resultant bleb rupture and pneumothorax formation.";"A mother brings her 35 year old son to the emergency department because of tremor and mutism for the past three days. His mother found him in his room this morning lying stiffly in his bed, soiled with urine and feces. He appears confused and will not respond to questions. He was diagnosed with schizophrenia last year and has been on several medications. Last month after his most recent hospital admission for schizophrenia, he was discharged with a prescription for haloperidol. On physical exam, he is visibly diaphoretic and has vital signs as follows: T 102.7, BP 140/98, P 112, R 12. His neuromuscular exam shows extremely rigid extremities, and his laboratory values are notable for a white blood cell count of 15000/mm3 and abnormally elevated creatine phosphokinase levels. What is the most likely explanation for these findings? A. neuroleptic-induced acute dystonia B. neuroleptic malignant syndrome C. schizophre - correct answer "B. neuroleptic malignant syndrome The answer is B. Neuroleptic malignant syndrome (NMS) is an idiosyncratic, life-threatening reaction to antipsychotic medications, with haloperidol being the most common cause. It is characterized by elevated temperatures, lead pipe"" muscle rigidity, altered mental status, choreoathetosis, tremors, and autonomic dysfunction (e.g., diaphoresis, labile blood pressure, incontinence, dysrhythmias). While this patient's temperature is only 102.7, students should note that any patients with temperatures greater than 105 most likely have non-infectious etiologies for temperature elevation. NMS is thought to be due to too much D2 blockade in the substantia nigra and hypothalamus. Treatment consists of stopping the causative agent and providing supportive care. Medications such as dantrolene, bromocriptine, amantadine, and lorazepam are also often used. Tardive dyskinesia (choice A) is a chronic movement disorder that results from prolonged use of antipsychotics and can include involuntary and periodic movements of the tongue or lips, mouth puckering, or flailing movements either of the extremities or of the spine. Neuroleptic-induced acute dystonia (choice C) is an acute spasm of a muscle or muscle group associated with the use of antipsychotic agents. It presents with patients complaining of neck twisting (torticollis), fixed upper gaze, facial muscle spasms, or dysarthria from tongue protrusions. In a similar family with dystonia, neuroleptic-induced akathisia (choice D) is an extrapyramidal syndrome that is manifest by agitation and restlessness. Schizophrenia, catatonic type (choice B), a diagnosis of exclusion, usually does not present with this degree of impairment.""";"A 25 year old man returns to the ED, 24 hours after SAEM EXAM QUESTIONS 2024-2025 ACTUAL EXAM REAL EXAM 1040 QUESTIONS AND CORRECT DETAILED ANSWERS WITH RATIONALES (VERIFIED ANSWERS) being released from the hospital with a new diagnosis of schizophrenia. He has recently started to take haloperidal for his psychotic symptoms. In the ED he is noted to have involuntary contractions of the muscles of the face, a protruding tongue, deviation of the head to one side, and sustained upward deviation of the eyes. Vital signs are stable, and initial labs show no electrolyte or hematological abnormalities. Of the following choices, the preferred medication for this condition is: A. diphenhydramine B. lorazepam C. phenobarbital D. metoprolol" - correct answer "A. diphenhydramine The answer is A. Acute dystonia, the most common adverse effect seen with neuroleptic agents, occurs in up to 5% of patients. Dystonic reactions, which can occur at any point during long-term therapy and up to 48 hours after administration of neuroleptics in the emergency department, involve the sudden onset of involuntary contraction of the muscles in the face, neck, or back. The patient may have protrusion of the tongue (buccolingual crisis), deviation of the head to one side (acute torticollis), sustained upward deviation of the eyes (oculogyric crisis), extreme arching of the back (opisthotonos), or rarely laryngospasm. These symptoms tend to fluctuate, decreasing with voluntary activity and increasing under emotional stress, which occasionally misleads emergency physicians to believe they may be hysterical in nature. Dystonic reactions should be treated with IM or IV benztropine (Cogentin®), 1 to 2 mg, or diphenhydramine (Benadryl®), 25 to 50 mg. Intravenous administration usually results in near- immediate reversal of symptoms. Patients should receive oral therapy with the same medication for 48 to 72 hours to prevent recurrent symptoms.";"A 70 year old male with acute delirium requires administration of haloperidol for agitation. Which of the following is a recognized side effect of haloperidol? A. first degree heart block B. nephrogenic diabetes insipidus C. prolonged QT interval D. transient hepatitis" - correct answer "C. prolonged QT interval SAEM EXAM QUESTIONS 2024-2025 ACTUAL EXAM REAL EXAM 1040 QUESTIONS AND CORRECT DETAILED ANSWERS WITH RATIONALES (VERIFIED ANSWERS) The answer is A. Excessive eye contact may be interpreted as a sign of aggression (answer A). Emergency physicians are encouraged to maintain intermittent eye contact with the patient and to keep a professional and calm demeanor. Also a physician should never deal with an agitated or violent patient alone in an isolated room (answer D). Doors should always remain open and exits should never be blocked. Ample security should be close at hand before interviewing the patient (answer E). Finally, involved parties are encouraged to remove any personal effects (e.g. neckties, necklaces, earrings, etc.) that could be used as a weapon by the violent patient.";"A 35 year-old male is placed on his back on the gurney in physical restraints for violent behavior. Which life-threatening complication can arise? A. circulatory obstruction B. Metabolic acidosis C. Asphyxia D. Rhabdomyolysis" - correct answer "B. Metabolic acidosis The answer is B. Bruises and abrasions are the most common complication of physical restraints. After restraint application, patients need to be monitored frequently and positions changed to prevent neurovascular complications such as circulatory obstruction, pressure sores, and rhabdomyolysis. Positional asphyxia can arise when patients are placed into the prone or hobbled position. Protracted struggle against restraints can promote a significant metabolic acidosis that has been associated with cardiovascular collapse. Patients who continue to struggle with physical restraints should be chemically restrained as well";"Which medication is ideal for the agitated or combative patient? A. Nitrous oxide B. Hydromorphone C. Haloperidol D. Propofol" - correct answer The answer is C. Drugs with a relatively short half-life allow for more careful monitoring of chemically restrained patients. Patients may be given multiple administrations of the restraining agent as needed. Antipsychotics (such as haloperidol) and benzodiazepines (such as lorazepam) exhibit most of these characteristics and are commonly used in combination in the emergency department. The use of 5 mg of haloperidol IV/IM with 2 mg of lorazepam IV/IM, repeated every 30 minutes as needed, is recommended for the combative patient who does not have contraindications to these medications. Half doses SAEM EXAM QUESTIONS 2024-2025 ACTUAL EXAM REAL EXAM 1040 QUESTIONS AND CORRECT DETAILED ANSWERS WITH RATIONALES (VERIFIED ANSWERS) should be used in the elderly.;"19 year old man is brought in to the emergency department by EMS after being found obtunded in his apartment by a friend. No additional history is available. On arrival, the patient is minimally responsive with sonorous respirations and a palpable rapid pulse. The most appropriate initial diagnostic test would be A. Arterial blood gas B. Electrocardiogram C. Fingerstick glucose D. Urine drug screen" - correct answer "C. Fingerstick glucose The answer is C. Hypoglycemia is a common and readily treatable cause for altered mental status. An ABG is unlikely to be diagnostic and more likely to reflect secondary abnormalities caused by respiratory depression. While a urine drug screen may show positives, it cannot quantitate the amount of a substance or the time period in which the exposure occurred so a positive screen may not reflect cause and effect. An EKG, while a part of a toxicology evaluation, is not an appropriate initiate screening test for an unstable patient until airway and readily reversible causes have been addressed.";"A 27 year old is found unresponsive in his car in the hospital parking lot and brought in by security. During your initial evaluation you find him to be cyanotic with pulse oximetry reading 82% on room air with a respiratory rate of 4 breaths per minute. Radial pulses are present at 120 bpm. Pupils are 1mm bilaterally. Your team is having difficulty finding a vein for an intravenous line due to extensive scarring of his arms. You are suspicious of an overdose, which medication would you want to rapidly administer as a potential antidote in this situation? A. Glucose B. Naloxone C. Thiamine D. Flumazenil" - correct answer "B. Naloxone The answer is B. The patient has stigmata of an opiate overdose with hypopnea, cyanosis, and miotic pupils. In addition, intravenous drug users often use up their veins. While hypoglycemia can definitely cause a depressed mental status and needs to be assessed, it should not result in respiratory depression or miotic pupils. Thiamine is utilized to prevent Wernicke's SAEM EXAM QUESTIONS 2024-2025 ACTUAL EXAM REAL EXAM 1040 QUESTIONS AND CORRECT DETAILED ANSWERS WITH RATIONALES (VERIFIED ANSWERS) encephalopathy particularly in malnourished patients who present with hypoglycemia but is not an antidote per se. Flumazenil can be used to temporarily reverse the respiratory depression caused by benzodiazepines but also carries with it the risk of precipitating withdrawal and uncontrollable seizures in chronic benzodiazepine users. As a result, it is not recommended for routine use in patients with altered mental status.";"A 53 year-old known alcoholic presents with agitation, vomiting and altered mental status. His fingerstick glucose is 148. His serum ethanol level is undetectable and his head CT is normal. An ABG shows a pH of 7.21, pCO2 of 34, pO2 of 98 on room air. His basic chemistry panel includes a sodium of 136, potassium 4.1, chloride 108, bicarbonate 14, BUN 12, creatinine 1.1. What substance are you concerned that he may have ingested A. Ethylene glycol B. Salicylates C. Isopropyl alcohol D. Methanol" - correct answer "C. Isopropyl alcohol The answer is C. The patient is presenting with a non anion gap metabolic acidosis. Isopropyl alcohol is metabolized via alcohol dehydrogenase to acetone which accumulates and causes significant ketosis but not an anion gap. Other toxic alcohols such as methanol and ethylene glycol are ultimately metabolized to formic and glycolic acids which cause toxic effects and an anion gap metabolic acidosis. Salicylates result in an anion gap metabolic acidosis with a superimposed respiratory alkalosis. The following mnemonic can be used to recall the common causes of an increased anion gap metabolic acidosis: CAT MUDPILES; C - cyanide A - alcoholic ketoacidosis T - toluene M - methanol U - uremia D - diabetic ketoacidosis P - paraldehyde I - isoniazid/iron SAEM EXAM QUESTIONS 2024-2025 ACTUAL EXAM REAL EXAM 1040 QUESTIONS AND CORRECT DETAILED ANSWERS WITH RATIONALES (VERIFIED ANSWERS) The answer is D. Alterations in mental status resulting from extreme emotional stimulus would usually be functional abnormalities. Patients with delirium manifest increases in alertness and psychomotor activity. Delirium is more than simple alteration of mental status. Delirium is an organic confusional state. Patients with delirium may have hallucinations, but patients who are oriented are more likely to have functional causes for altered mental status.";"Which of the following statements regarding psychotic behavior is true? A. Brief psychotic episodes, often precipitated by events such as death of a loved one, can be characterized by extremely bizarre behavior and speech B. Delusions are defined as false beliefs that are not amenable to arguments or facts to the contrary C. Delusional disorder usually results in impairment in daily functioning D. Schizophreniform disorder is present when a patient meets the diagnostic criteria for schizophrenia but the process has been present for less than one year" - correct answer "B. Delusions are defined as false beliefs that are not amenable to arguments or facts to the contrary B. Psychosis can be limited to nonbizarre delusions; patients with this disorder (delusional disorder) rarely have impairment in daily functioning. Fixed, false beliefs that are not held by others with a patient's cultural background are characteristic of delusional thinking.";"A 75 year old female is brought the to emergency department by a family member with a history of progressive forgetfulness and confusion. She has a history of dementia. The most common cause of dementia in the elderly patient is: A. Alzheimer's disease B. Parkinson's disease C. Pick's disease D. Vascular dementia" - correct answer A. Most dementia is Alzheimer's type. The second most common cause of dementia is vascular dementia, which accounts for 10 to 20% of all dementias. Primary degenerative dementias include Alzheimer's disease, vascular dementia, subcortical dementias involving the basal ganglia and thalamus (e.g., progressive supranuclear palsy, Huntington's chorea, Parkinson's disease), and Pick's disease, also known as dementia of the frontal lobe type. Smaller percentages are attributable to causes such as anoxic encephalopathy, hepatolenticular degeneration, tumors, and slow virus infections.;"A 65 year old male is brought to the emergency department after he was found wandering on the street. SAEM EXAM QUESTIONS 2024-2025 ACTUAL EXAM REAL EXAM 1040 QUESTIONS AND CORRECT DETAILED ANSWERS WITH RATIONALES (VERIFIED ANSWERS) He is unkempt and confused. A diagnosis of delirium, rather than dementia, is more likely if which of the following is true? A. there has been a slow progressive loss of memory B. the sleep-wake cycle is unaffected C. there is a change in the level of consciousness D. the confusion is worse during the day" - correct answer "C. there is a change in the level of consciousness The answer is C. Patients with delirium have disturbances in consciousness, cognition, and perception. These disturbances tend to occur over a short period of time (hours to days). The delirious patient may be somnolent or agitated. Thought process may be mildly disturbed or grossly disorganized. The clinical presentation may be subdued or explosive, and the course can fluctuate over minutes to hours. The patient's sleep-wake cycle may be altered or reversed; agitation is often present during the night. An acute confusional state can also be one of the protean manifestations of a metabolic or nutritional abnormality, including hepatic encephalopathy, acute renal failure, and diabetic ketoacidosis or hyperosmolarity.";"An 80 year old nursing home patient is brought to the emergency department with an acute onset of confusion. Which of the following metabolic abnormalities is the most likely explanation? A. hypocalcemia B. hypokalemia C. hypernatremia D. hyperphosphatemia" - correct answer "C. hypernatremia The answer is C. The differential diagnosis of acute confusional states is lengthy. It includes many metabolic/nutritional abnormalities including hypoglycemia, hypo-/hypernatremia, and hypercalcemia. Hypokalemia alone, however, is not a common cause of altered mental status.";"Which factor is least reliable in differentiating between organic and inorganic causes of confusion? A. acute versus chronic onset B. Vital sign abnormalities C. Presence of attention deficit SAEM EXAM QUESTIONS 2024-2025 ACTUAL EXAM REAL EXAM 1040 QUESTIONS AND CORRECT DETAILED ANSWERS WITH RATIONALES (VERIFIED ANSWERS) D. Signs of trauma" - correct answer "C. Presence of attention deficit The answer is C. Presence of an attention deficit is common to all confusional states. All the other options may be used to differentiate organic versus non-organic causes of confusion. Characteristics of organic causes include acute onset, abnormal vital signs, fluctuating level of consciousness, possibly signs of trauma, and/or focal neurologic signs. Inorganic (functional) causes commonly illustrate chronic onset, stable vital signs, absence of trauma or focal neurologic symptoms, and/or delusions and illusions.";"With regard to specific causes of hypertension, which of the following is true? A. Hypertensive encephalopathy is more likely than hypertensive stroke in patients whose mental status changes are reversible B. Hypertensive encephalopathy causes adverse outcomes over days or weeks, rather than hours C. Patients with stroke syndromes must have blood pressure normalized as quickly as possible to reduce the risks of worsening neurological deficit D. Laboratory analysis is rarely useful in cases of confirmed pediatric hypertension E. Laboratory analysis is rarely useful in cases of confirmed hypertension in pregnant patients" - correct answer "A. Hypertensive encephalopathy is more likely than hypertensive stroke in patients whose mental status changes are reversible The answer is A. Hypertensive encephalopathy is a true medical emergency, and can cause coma and death over hours; however, encephalopathy due to hypertension is more likely reversible than encephalopathy from other causes. Avoidance of overzealous blood pressure lowering is particularly critical for patients with strokes. Laboratory analysis can be important in cases of hypertension in pediatric patients (for whom renal/renovascular or pheochromocytoma may be identified) and in pregnant patients (for whom laboratory testing can help establish diagnoses such as the HELLP syndrome).";"A 29 year old woman is found seizing by her husband and is rushed to the emergency department. On presentation, she is noted to have a BP of 162/112, is still seizing, and looks puffy all over. Her husband tells you that they are expecting their first child in a few months. Which of the following is the next best step in this patient's care? A. Control the seizures with magnesium sulfate. B. Draw blood to check CBC, LFT's, BUN, and creatinine. SAEM EXAM QUESTIONS 2024-2025 ACTUAL EXAM REAL EXAM 1040 QUESTIONS AND CORRECT DETAILED ANSWERS WITH RATIONALES (VERIFIED ANSWERS) elderly is medications, accounting for 22-39% of cases. Infection and metabolic abnormalities are other common causes, and delirium may be the first indication that an infection is present. An elderly patient with delirium resulting from an infection may have a normal temperature, a low temperature, or a high temperature. Delirium is characterized by an acute onset of a disturbance in level of consciousness, cognition and attentiveness. It has a fluctuating course, and alterations in sleep-wake cycles are common. Dementia, in contrast, has a slower course, that is gradually progressive over months to years, and consciousness is preserved. In addition to correcting the underlying cause, it is important to minimize stimulation, because the patient with delirium has difficulty processing stimuli.";"The chest X-ray in the Figure was taken in an intoxicated patient who is conversant, but an unreliable historian. The X-ray findings are best described as indicating: [image] A. Esophageal foreign body B. Intratracheal foreign body C. mediastinitis from esophageal perforation D. normal chest" - correct answer "A. Esophageal foreign body The answer is A. The film reveals a classic appearance of a round foreign body (in this case, a pull-top from a beer can) in the esophagus. The foreign body appears to lie outside the tracheal shadow. There is no sign of mediastinal air (which would be expected with penetrating trauma). The X-ray reveals no signs of mediastinitis, but the risk of esophageal perforation and ultimate mediastinitis prompts endoscopic intervention in this patient.";"An 18 year old hockey player is hit in the mouth with a puck, fracturing a maxillary canine tooth. He brings the severed piece of tooth with him. On physical exam, the tooth is fractured halfway between the tip and the gumline. The root of the tooth is still firmly intact. The exposed fracture site has a yellowish tinge without blood. Of the following choices, which is the most appropriate management for this patient? A. No specific treatment required B. Application of calcium hydroxide, placement of aluminum foil, and dental follow-up C. Placement of tooth fragment in saline gauze, outpatient dental follow-up D. Immediate dental consult to avoid abscess formation SAEM EXAM QUESTIONS 2024-2025 ACTUAL EXAM REAL EXAM 1040 QUESTIONS AND CORRECT DETAILED ANSWERS WITH RATIONALES (VERIFIED ANSWERS) E. Replace fractured piece and place acrylic splint" - correct answer "D. Immediate dental consult to avoid abscess formation The answer is D. Ellis II dental fracture involves enamel and dentin. The fracture site typically has a yellowish tinge. Ellis III dental fractures are characterized by exposure of pinkish pulp and often blood. These fractures require immediate dental consultation to prevent abscess formation.";"A 22 year old man is punched in the nose during a fight. He presents to the emergency department with obvious nasal bone deformity. Pressure controls the bleeding. Physical exam reveals no maxillary bone or orbital rim tenderness, intact vision and extraocular movement. The oropharynx and mandible are unremarkable. Nasal inspection reveals a swollen, ecchymotic, tender nasal septum. Which of the following is the most appropriate initial step? A. Outpatient follow-up with an ENT specialist to surgically correct a deviated septum B. Plastic surgery consult for immediate reduction of nasal fracture C. Facial CT scan to rule out more serious facial fractures D. Incision and drainage of the septal hematoma followed by nasal packing E. Needle aspiration of the septal hematoma" - correct answer "D. Incision and drainage of the septal hematoma followed by nasal packing";"A 24 year old woman is playing racquetball and sustains a direct blow from the ball to the right eye. She presents to the emergency department complaining of eye pain and double vision. On exam, her right eye does not track properly with upward gaze. This finding suggests which of the following injuries? A. Inferior orbital wall fracture B. Superior orbital rim fracture C. Ethmoid fracture D. Zygomatic arch fracture E. Inferior orbital rim fracture" - correct answer "A. Inferior orbital wall fracture The answer is A. The patient most likely has an orbital floor fracture with entrapment.";"A 32 year old man is struck several times in the head with a baseball bat. Upon emergency medical service arrival, he is mildly confused, vomits once, and complains of a severe headache. The emergency medical technicians establish two large-bore IVs. Prior to arrival at the emergency SAEM EXAM QUESTIONS 2024-2025 ACTUAL EXAM REAL EXAM 1040 QUESTIONS AND CORRECT DETAILED ANSWERS WITH RATIONALES (VERIFIED ANSWERS) department, he loses consciousness and begins to seize. He is actively seizing when he is brought into the trauma bay. What should be the first step in the management of this patient? A. Administration of phenytoin 1000mg IV B. Administration of mannitol 50 g IV C. Rapid sequence intubation using paralytic agent D. Emergency craniotomy E. Administration of 2 liters NS bolus" - correct answer "C. Rapid sequence intubation using paralytic agent The answer is C. The airway should be managed as the first priority in this patient. The other maneuvers may be helpful but are secondary to securing an airway and providing oxygenation/ventilation. Airway comes first!";"A 46 year old man is brought in by EMS after a motor vehicle collision in which he was an unrestrained driver. Although he has no obvious injury to his head or neck, he complains of chest pain and appears very short of breath. His vital signs are: T 99.2 F, BP 85/57, HR 123, RR 36, SpO2 95% on non-rebreather. The CXR demonstrates a tension pneumothorax. Of the following, which is the most appropriate next step in this man's care? A. Placement of a chest tube followed by a chest xray to determine proper placement B. Transfusion of 2 units of O-negative packed red blood cells C. Performance of a chest CT scan to further delineate the pathology D. Placement of a needle decompression device, followed by repeat CXR" - correct answer "D. Placement of a needle decompression device, followed by repeat CXR This patient needs emergent chest decompression and this is rapidly done by needle thoracostomy. A chest CT may be performed, but only once he is stabilized. A formal chest tube will be placed, but placement may not be rapid enough and he may decompensate in the meantime. Transfusion of blood does nothing to correct the physiology of a tension pneumothorax";"The most sensitive bedside test for nerve injury in a finger after trauma is: A. light touch SAEM EXAM QUESTIONS 2024-2025 ACTUAL EXAM REAL EXAM 1040 QUESTIONS AND CORRECT DETAILED ANSWERS WITH RATIONALES (VERIFIED ANSWERS) D. orbital blow-out fracture E. ruptured globe" - correct answer "A. retrobulbar hematoma The answer is A. Traumatic proptosis with impaired extraocular movements is classic for retrobulbar hematoma. Sequelae include optic nerve ischemia and secondary visual impairment. A ruptured globe presents with enophthalmos, not proptosis, as vitreous humor leaks out of the eye. Neither hyphema nor chemosis causes proptosis. Orbital blowout fractures can cause inferior rectus muscle entrapment and secondary pain with impairment of extraocular movement. Yet, they do not present with proptosis - unless complicated by retrobulbar pathology.";"Following a brawl at a local bar, a gentleman presents with an impressive right-sided periorbital ecchymosis. All of the following physical examination findings would suggest an orbital blowout fracture EXCEPT: A. diplopia with upward gaze B. right-sided infraorbital subcutaneous emphysema C. right-sided epistaxis D. proptosis E. anesthesia of the right infraorbital region" - correct answer "D. proptosis D. Orbital blowout fractures classically involve the maxillary or ethmoid sinus and consequently often cause either epistaxis (through the connection of the maxillary sinus with the nose) or subcutaneous emphysema (through the entry of air from the sinuses into the subcutaneous tissue). A fracture through the maxillary sinus may extend through the portal by which the second branch of the trigeminal nerve exits, thus causing anesthesia of the ipsilateral infraorbital region. If the inferior rectus muscle gets trapped within the fracture of the inferior orbital wall, patients will be unable to look upward causing diplopia with upward gaze. Orbital blowout fractures are not typified by proptosis. In fact, proptosis in the setting of trauma should prompt physicians to suspect the possibility of a retrobulbar hematoma.";"The patient depicted in the figure presents to the ED just after sustaining a pellet-gun wound to the right eye. What do the arrows most likely indicate? [image] A. hyphema B. iritis SAEM EXAM QUESTIONS 2024-2025 ACTUAL EXAM REAL EXAM 1040 QUESTIONS AND CORRECT DETAILED ANSWERS WITH RATIONALES (VERIFIED ANSWERS) C. keratitis D. pterygium E. hypopion" - correct answer "A. hyphema The answer is A. The patient has a fluid level/meniscus in the anterior chamber, that is most likely indicative of hyphema (collection of blood). Hypopion (collection of purulent material) is less likely in this setting, and keratitis (corneal inflammation) and iritis (inflammation of the iris) are not indicated by the arrows. A pterygium is a growth which is visible on the sclera, and which crosses the limbus onto the cornea.";"The patient in the figure sustained minor blunt trauma to the eye, and has a normal head/orbital computed tomography (CT) scan. Ophthalmological examination is normal, other than the blood as shown in the figure (the blood does not cross the limbus). Of the choices below, which diagnosis is the most likely based upon the figure? [image] A. foreign body B. ruptured anterior chamber C. subconjunctival hemorrhage D. hyphema E. globe rupture" - correct answer "C. subconjunctival hemorrhage The answer is C. The subconjunctival blood as depicted in this patient, can be expected to resorb without intervention over days to weeks. The figure does not suggest rupture of anterior chamber or hyphema; globe rupture and foreign body are less likely given the normal examination and CT scan.";"A patient presents with a self-inflicted wound, with resultant loss of vision in the right eye. With regard to the figure, which of the following statements is most likely true? [image] A. Medial canthotomy should be performed immediately. B. If ambulance providers contact medical control about a patient with this injury, they should be directed to replace the globe back into the orbit. SAEM EXAM QUESTIONS 2024-2025 ACTUAL EXAM REAL EXAM 1040 QUESTIONS AND CORRECT DETAILED ANSWERS WITH RATIONALES (VERIFIED ANSWERS) C. Search for other self-inflicted injuries (or ingestions) is paramount. D. Life-threatening hemorrhage is a major risk with this injury. E. The patient will probably recover visual function." - correct answer "C. Search for other self- inflicted injuries (or ingestions) is paramount. The answer is C. The patient is unlikely to recover any visual function, which renders more important parallel efforts to identify (treatable) injuries or ingestions that are less obvious than the ocular avulsion. Lateral (not medial) canthotomy is an emergency procedure that may be indicated in some patients with ocular injury and retrobulbar hematoma (with resultant traction on the optic nerve), but the procedure is unlikely to help this patient. Since manipulation of an injured globe risks further trauma and extrusion of vitreous humor, prehospital recommendations for eye trauma are limited to protection of the injured eye and expedited transport to definitive care. Life-threatening hemorrhage is not a major risk with this type of injury.";"What is the most common cause of death in Americans aged 20 to 40 years? A. Drug overdose B. Trauma C. Cancer D. AIDS-related illness E. Pneumonia" - correct answer "B. Trauma";"Following a motor vehicle collision, in which of the following patients is an emergency department Caesarian section most likely indicated, assuming a fetus at 29-weeks gestation? A. Mother with gunshot wound to abdomen, blood pressure 96/42; fetal heart tones undetectable B. Mother with severe head trauma, blood pressure 170/90; fetal heart tones 120 beats per minute C. Mother with abdominal pain, blood pressure of 80/40; fetal heart tones 100 beats per minute D. Mother with vaginal bleeding, blood pressure 118/78; fetal heart tones 80 beats per minute E. Mother pulseless and apneic for 2 minutes' duration and still in arrest; fetal heart tones of 100 beats per minute" - correct answer "E. Mother pulseless and apneic for 2 minutes' duration and still in arrest; fetal heart tones of 100 beats per minute SAEM EXAM QUESTIONS 2024-2025 ACTUAL EXAM REAL EXAM 1040 QUESTIONS AND CORRECT DETAILED ANSWERS WITH RATIONALES (VERIFIED ANSWERS) A. 22 year old with stab wound to fourth intercostal space on right B. 30 year old with pelvic pain and tenderness after fall C. 25 year old restrained passenger in high-speed MVA; no abdominal complaints D. None of the above E. All of the above" - correct answer "E. All of the above The answer is E. The diaphragm can rise to as high a level as the fourth intercostal space and can be injured by stab wounds at this level. Unsuspected injuries are common in high speed motor vehicle crashes. Pelvic injuries are associated with intra-abdominal injuries and can distract a patient from such an injury. Therefore, all of these patients need an evaluation of their abdomens.";"Which of the following trauma patients can be managed conservatively without immediate laparotomy in the OR? A. 27 year old man with hemoperitoneum by bedside ultrasound; hypotensive B. 19 year old man with splenic laceration; peritoneal signs on exam C. 24 year old man with liver laceration; hemodynamically stable D. 30 year old man with a gunshot wound to the epigastrium E. All of the above should go to the OR for exploratory laparotomy." - correct answer "C. 24 year old man with liver laceration; hemodynamically stable";"Which of the following is an accurate statement? A. Bedside ultrasound is the test of choice for diagnosing solid organ injury. B. Diagnostic peritoneal lavage usually cannot identify the presence of hemoperitoneum. C. Bedside ultrasound can image the retroperitoneum. D. Bedside ultrasound can reliably determine the etiology of hemoperitoneum. E. Diagnostic peritoneal lavage cannot determine the etiology of hemoperitoneum." - correct answer "E. Diagnostic peritoneal lavage cannot determine the etiology of hemoperitoneum. The answer is E. Diagnostic peritoneal lavage is extremely sensitive for the detection of hemoperitoneum and can lead to many negative laparotomies. Neither bedside ultrasound nor diagnostic peritoneal lavage can identify the source of the hemorrhage though. A trauma SAEM EXAM QUESTIONS 2024-2025 ACTUAL EXAM REAL EXAM 1040 QUESTIONS AND CORRECT DETAILED ANSWERS WITH RATIONALES (VERIFIED ANSWERS) ultrasound at the bedside can only identify fluid in the peritoneal cavity, and CT scan is the test of choice for diagnosing solid organ injury.";"A 36 year old man is a restrained driver involved in a high speed MVA where his car is struck on the driver's side door with significant intrusion. His physical exam is significant for a large contusion on his left flank. His abdominal exam is benign and rectal exam reveals a normal prostate. A Foley catheter is placed with return of gross hematuria. Which test is indicated to evaluate for the presence of urologic injury? A. CT abdomen / pelvis with IV and transurethral contrast B. Ultrasound of the bladder C. CT abdomen / pelvis without contrast D. Ultrasound of the kidneys E. CT abdomen / pelvis with IV contrast alone" - correct answer "A. CT abdomen / pelvis with IV and transurethral contrast";"In which of these patients is emergency department thoracotomy indicated? A. All of the above should undergo emergency department thoracotomy. B. Unbelted driver in a high-speed motor vehicle crash who loses his pulse while being extricated, and arrives at the E.D. after a 45-minute transport C. Patient with stab wound to the anterior chest who is dyspneic with an oxygen saturation of 80% and a blood pressure of 168/102 D. Pedestrian struck with massive pelvic fractures who loses pulses and blood pressure at the scene E. Patient with a gunshot wound to the chest who upon arrival is unconscious and pulseless, with a systolic blood pressure of 60" - correct answer "E. Patient with a gunshot wound to the chest who upon arrival is unconscious and pulseless, with a systolic blood pressure of 60 The answer is E. Emergency Department thoracotomy is a controversial procedure. When chosen carefully, successful resuscitation can occur. Cardiac arrest due to blunt trauma has a dismal success rate and is generally not considered an indication for ED thoracotomy. Thoracotomy for penetrating chest wounds has the best success rate. An awake patient with a relatively normal blood pressure does not need one performed in the Emergency Department. An unconscious and pulseless patient with a detectable blood pressure has the best chance for survival.";"A 32 year old female is shot with a 38-caliber pistol at close range in the right anterior chest. She presents to the emergency department intoxicated and yelling. Her vitals SAEM EXAM QUESTIONS 2024-2025 ACTUAL EXAM REAL EXAM 1040 QUESTIONS AND CORRECT DETAILED ANSWERS WITH RATIONALES (VERIFIED ANSWERS) include a pulse of 92, blood pressure of 134/84, and oxygen saturation of 97%. She has clear breath sounds bilaterally. The entrance wound is just above the right breast and an exit wound is noted in the right axilla. What is the most appropriate management of this patient? A. IV access, endotracheal intubation and simultaneous placement of a right chest tube, bedside ultrasound, portable chest X-ray, and admission to the ICU if stable B. IV access, portable chest X-ray, tube thoracostomy, and exploratory thoracotomy in the OR to search for cardiac or pulmonary vascular injury C. IV access, endotracheal intubation, emergency department thoracotomy to search for cardiac or pulmonary vascular injury D. IV access, portable chest X-ray, right c - correct answer "A. IV access, endotracheal intubation and simultaneous placement of a right chest tube, bedside ultrasound, portable chest X-ray, and admission to the ICU if stable";"During a bar fight, a 42 year old man is stabbed in the left side with an unknown weapon. He presents to the emergency department with dyspnea, pulse of 108, blood pressure of 138/92, and oxygen saturation of 94% on room air. He has absent breath sounds on the left side; you note a small puncture wound in the midaxillary line at the level of the 10th rib. His abdominal exam is normal. Two large-bore IVs are established. What is the appropriate management of this patient? A. Left-sided chest tube, portable chest x-ray, and admission B. Left-sided chest tube, portable chest x-ray, diagnostic peritoneal lavage, and admission C. Endotracheal intubation, left-sided chest tube, portable chest x-ray, and admission D. Endotracheal intubation, portable chest x-ray, exploratory laparotomy in the OR, and admission E. Left-sided chest tube, portable chest x-ray, and abdominal CT scan" - correct answer "E. Left-sided chest tube, portable chest x-ray, and abdominal CT scan";"Which is the most common associated neurological finding with a distal radius fracture? A. Weakness with flexion at the finger MCP joints B. Wrist drop C. Decreased sensation over the hypothenar eminance D. Weakness of finger adduction E. Decreased sensation over the thenar eminance" - correct answer "E. Decreased sensation over the thenar eminance SAEM EXAM QUESTIONS 2024-2025 ACTUAL EXAM REAL EXAM 1040 QUESTIONS AND CORRECT DETAILED ANSWERS WITH RATIONALES (VERIFIED ANSWERS) B. If it is suspected that the two wounds are from the same missile, the emergency department physician's documentation should note which wound is the entrance, and which is the exit C. Wound description is essential for the emergency medicine specialist. However, description of a wound as to entrance or exit is best left to foren - correct answer "C. Wound description is essential for the emergency medicine specialist. However, description of a wound as to entrance or exit is best left to forensic examination. The answer is C. The overall evidence points to a superior-medial to inferior-lateral wound trajectory, with subcutaneous ecchymosis indicating the missile track and the more ragged wound at the inferior-lateral (groin) region most likely an exit wound. However, though wound description is very important for the emergency physician (both as a guide to injury evaluation and also as an early characterization of wounds, before interventions such as wound exploration obscure physical findings), speculation as to whether wounds are entrance or exit wounds are best left off of the E.D. record. Clinicians tend to oversimplify and/or misinterpret physical wound characteristics. Thus, the best course is a meticulous description (or photograph) of the wound, noting items such as tattooing (i.e. of gunpowder) or stellate tissue destruction (which can be due to expansion of gun barrel gases in a contact wound) but leaving interpretation of the physical evidence to forensics experts. The wound characteristics are not consistent with self-inflicted injury, though the ED physician should have a low index of suspicion for psychiatric consultation when there is doubt on this subject.";"A patient presents to the ED after a fall with chest pain. A chest xray shows a rib fracture but no pneumothorax, and a chest CT is ordered. What is the most appropriate treatment for a small pneumothorax, detected only on chest CT, in a hemodynamically stable trauma patient? A. 100% oxygen B. Heliox by face mask C. Immediate needle decompression D. Chest tube placement" - correct answer "A. 100% oxygen The answer is A. An occult pneumothorax may resorb with only oxygen administration, not requiring invasive management. Needle decompression is used for tension pneumothorax, and heliox may be used for reactive airway disease to reduce resistance to flow.";"A blunt trauma patient presents, transported by EMS from a motor vehicle collision, with inability to provide a history, due to alcohol intoxication. He has no signs of trauma on external evaluation, but he is SAEM EXAM QUESTIONS 2024-2025 ACTUAL EXAM REAL EXAM 1040 QUESTIONS AND CORRECT DETAILED ANSWERS WITH RATIONALES (VERIFIED ANSWERS) hypotensive. An ED ultrasound is performed at the bedside, and is depicted in the figure. Given the patients' clinical condition and image seen, what is the most likely diagnosis? [image] A. Rupturing abdominal aortic aneurysm B. Free intraabdominal fluid C. Ruptured gallbladder D. Fat embolus from femur fracture" - correct answer "B. Free intraabdominal fluid";"You are practicing in a trauma center a receive a call from an outlying facility that they would like to transfer a male patient to you with a spinal cord injury after significant flexion and compression of the vertebral body. What does this injury pattern tell you about the patient's symptoms? [image] Figure used with permission from Hamilton et al, Emergency Medicine: An approach to clinical problem-solving A. The patient likely has symptoms on only one side of the his body B. Patients with anterior cord syndromes have only sensory symptoms C. The patient likely disproportionately greater weakness in the lower extremities (as compared to the upper extremities) D. The patient likely has paralysis and loss of sensation to pain and temperature bilaterally below the lesion" - correct answer "D. The patient likely has paralysis and loss of sensation to pain and temperature bilaterally below the lesion The answer is D. Answer A describes central cord syndrome, typically caused by hyperextension. Answer C describes Brown-Sequard Syndrome, caused by hemisection of the cord. Answer B is anterior cord, often caused by flexion and injury to the anterior spinal artery; patient with this cord syndrome often have more than just sensory symptoms.";"A trauma patient resuscitated in the ED, has a post-tube thoracostomy computed tomography (CT) scan as depicted in the figure. What finding is present? [image] SAEM EXAM QUESTIONS 2024-2025 ACTUAL EXAM REAL EXAM 1040 QUESTIONS AND CORRECT DETAILED ANSWERS WITH RATIONALES (VERIFIED ANSWERS) A. Pericardial tamponade B. Aortic rupture leading to a right hemothorax C. Chest tube not within the thoracic cavity D. Persistent pneumothorax" - correct answer "D. Persistent pneumothorax The answer is D. The chest CT shows a chest tube in place on the right, with incomplete reinflation of the lung. The left hemithorax is grossly normal, but there is a large right pulmonary contusion, as well as pneumothorax and small hemothorax on the right. The aorta appears intact (aortic injuries usually leak into the left hemithorax, not the right).";"he major abnormality on the image below is at which level? [image] A. C3/C4 B. C4/C5 C. C5/C6 D. C6/C7" - correct answer "C. C5/C6 he answer is C. The patient has anterior displacement of C5 on C6.";"In differentiating high voltage electrical injury from lightning injury, which of the following is your best discriminator? A. Deep burns B. Fractures or dislocations C. Loss of consciousness D. Cardiac arrest" - correct answer "A. Deep burns The answer is A. Patients with high voltage injury commonly present with devastating burns. The burns are most severe at the source and ground contact points. The most common sites of contact with the source include the hands and the skull. The most common areas of ground SAEM EXAM QUESTIONS 2024-2025 ACTUAL EXAM REAL EXAM 1040 QUESTIONS AND CORRECT DETAILED ANSWERS WITH RATIONALES (VERIFIED ANSWERS) herself by jumping out a window. She has a long history of depression. Regarding suicide, which of the following statements is FALSE? A. The majority of suicide attempts involve minor injuries or drug overdoses. B. All states have laws giving law enforcement the right to place into custody any individual suspected of being a danger to themselves or others. C. The patient's room needs to be cleared of all potentially dangerous objects such as blunt instruments, glass objects, and the patient's belongings. D. Family sitters provide the best option for close observation of suicidal patients since they often have a calming influence on the patient." - correct answer "D. Family sitters provide the best option for close observation of suicidal patients since they often have a calming influence on the patient. The answer is D. For any patient presenting with suicidal ideation, the emergency department physician must first stabilize medical condition as most attempts involve minor injury or drug overdoses treatable by emergency room staff. Secondly, all objects and substances should be kept strictly out of reach of the suicidal patient. Once stabilized these patients may be kept under direct supervision. It is not recommended that family members provide the direct supervision because of a possibility of collaboration between family and patient to leave the hospital. In this event, or even before coming into the hospital, all law enforcement has the right to bring into custody any patient at risk of harming self or others.";"Suicide rates are consistent with the average population in this population: A. Presence of underlying panic disorder B. African-American males C. Men with AIDS D. Females with breast implants" - correct answer "B. African-American males The answer is B. The epidemiology of suicide varies according to age, gender and race. Overall, men have higher rates of completed suicide and women have greater numbers of suicide attempts. Suicide risk among men shows a bimodal distribution with increased risk between the ages of 15 and 24 and after age 65. Suicide risk among women peaks after age 60. People who suffer from a chronic disease such as AIDS are at increased risk for committing suicide. The dramatic increase in suicide rates among adolescents is thought to be due to changing SAEM EXAM QUESTIONS 2024-2025 ACTUAL EXAM REAL EXAM 1040 QUESTIONS AND CORRECT DETAILED ANSWERS WITH RATIONALES (VERIFIED ANSWERS) demographics and greater access to firearms. Underlying psychiatric illness increases a person's risk of suicide. Major depression, bipolar, borderline personality disorder, schizophrenia, and panic disorder are all associated with increased suicide rates. Of note, One study demonstrated that 40% of people who suffer from panic attacks would attempt suicide at some point in their lives. Studies have demonstrated that the presence of breast implants portends a higher risk of suicide.";"Which clinical scenario is use of chemical restraint indicated? A. Patient's remarks are felt to warrant negative reinforcement B. Patient's behaviors and actions pose an imminent harm to self C. Patient is responding to verbal tactics, but requires multiple attempts D. Patient is uncooperative with the history" - correct answer "B. Patient's behaviors and actions pose an imminent harm to self he answer is B. Physical and chemical restraints should only be used when verbal tactics have failed and when the patient is an immediate threat to himself, others or the integrity of the emergency department. From a medico-legal standpoint, clear documentation is very important when a decision is made to apply restraints. Answers A, C, D, and E are all valid reasons for physical or chemical restraints.";"Which of the following psychiatric disorders is associated with the greatest increased risk of committing suicide? A. post-traumatic stress disorder (PTSD) B. schizophrenia C. panic disorder D. major depression" - correct answer "C. panic disorder The answer is C. Most people who commit suicide suffer from either alcoholism or a diagnosable psychiatric illness. 15-20% of people with major depression and 10% of people with schizophrenia will commit suicide. Up to 40% of people with panic disorder will attempt suicide at some point in their lives. PTSD also carries an increased risk.";"When using the "SAD PERSONS" scale to determine suicide risk, which factor conveys the least amount of points? A. depression or hopelessness B. rational thinking loss C. separated, divorced or widowed SAEM EXAM QUESTIONS 2024-2025 ACTUAL EXAM REAL EXAM 1040 QUESTIONS AND CORRECT DETAILED ANSWERS WITH RATIONALES (VERIFIED ANSWERS) D. stated future intent" - correct answer "C. separated, divorced or widowed The answer is C. Being separated, divorced or widowed is an important but less significant factor in determining suicide risk and so is assigned 1 point on the suicide scale. All the others are high-risk factors and are each assigned 2 points on the suicide scale. A score of 6 or more has a sensitivity of 94% and a specificity of 71% compared with formal psychiatric evaluation to identify the need for hospitalization in patients who present immediately after a suicide attempt.";Silent Suicide" is defined as: - correct answer ; - correct answer A.; - correct answer B.; - correct answer C.; - correct answer D.; - correct answer E.;"Toxicological screening is indicated in which patient with suicidal ideation? A. Patient who ingested unknown amount ibuprofen 48 hours earlier B. Patient who threatens to cut both wrists with a knife C. Patient who takes lithium for bipolar affective disorder D. Patient who ingested a "bottle" of tylenol" - correct answer "D. Patient who ingested a "bottle" of tylenol The answer is D. Routine toxicological screening is unnecessary in the evaluation of suicidal patients in whom there are no clinical indications for such testing. With the exception of acetaminophen, essentially all patients with dangerous overdoses and poisoning will demonstrate clinical signs within several hours of ingestion. History, physical examination, and risk determination of suicide, however, is part of the routine evaluation of the suicidal patient.";"Suicide risk is increased in this patient population: A. Patients who are elderly and Caucasian B. Patients who have not been involuntarily committed C. Patients who directly questioned about suicide D. Patient who takes lithium for bipolar affective disorder" - correct answer "A. Patients who are elderly and Caucasian The answer is A. TRoutine toxicological screening is unnecessary in the evaluation of suicidal patients in whom there are no clinical indications for such testing. With the exception of acetaminophen, essentially all patients with dangerous overdoses and poisoning will SAEM EXAM QUESTIONS 2024-2025 ACTUAL EXAM REAL EXAM 1040 QUESTIONS AND CORRECT DETAILED ANSWERS WITH RATIONALES (VERIFIED ANSWERS) removal more difficult and increases the risk of pressure necrosis. Irrigation should also be avoided in cases of tympanic membrane perforation. -- For further reading see Rosen's Emergency Medicine: Concepts and Clinical Practice, 5th edition, Chapter 53. The answer is B. A, C, D, and E are all acceptable means of foreign body removal from the ear canal. Irrigating the ear canal with warm water is acceptable in many instances, and may remove a foreign object with minimal discomfort. Direct the water jet past the object, against the tympanic membrane, and then back out the ear canal, hopefully dislodging the foreign object in the process. However, this technique should not be used in cases of bean or seed insertion because such objects swell when moistened, which makes subsequent removal more difficult and increases the risk of pressure necrosis. Irrigation should also be avoided in cases of tympanic membrane perforation.";"You suspect that your patient has swallowed a nail. Which of the following is an indication for endoscopic or surgical removal of this object? A. Abdominal CT scan shows a 1cm nail in the distal sigmoid colon B. Plain films do not reveal a radiopaque foreign body in the chest or abdomen C. Radiography visualizes the nail in the gastric fundus D. The object has progressed from the jejunum through the ileum after 24 hours" - correct answer "C. Radiography visualizes the nail in the gastric fundus The answer is C. Observation can manage most ingested foreign objects - the patient should pass the object after several days. Propulsive agents can be given to speed movement along the gastrointestinal tract. Serial radiography monitors movement. Endoscopic or surgical intervention is indicated for sharp objects (which may cause perforation), objects greater than 2 cm in width (which are likely to lodge at the pylorus or the ileocecal valve), and long rigid objects (which may have trouble passing through the right angles of the duodenum). Surgery is indicated if an object fails to move after 24 hours (indicating impaction), or if the patient develops symptoms of obstruction or perforation. Body packers may have ingested substances such as cocaine or heroin, which can cause great harm should the packaging be disrupted. Proper management is observation, as most will pass the package(s) without complications. Urgent package retrieval should not be performed because of pressures from law enforcement. Endoscopy can in fact be dangerous to the patient, as it can disrupt the packaging and release toxic drugs. Endoscopic or surgical intervention is warranted should the patient develop signs of systemic drug toxicity. The patient SAEM EXAM QUESTIONS 2024-2025 ACTUAL EXAM REAL EXAM 1040 QUESTIONS AND CORRECT DETAILED ANSWERS WITH RATIONALES (VERIFIED ANSWERS) can also be monitored with serial serum drug levels.";"A 30-year old female, without past medical history, presents with "an ingrown hair" in her thigh, as depicted in the Figure. She is afebrile, nontoxic, and has no regional lymphadenopathy or lymphangitis. Examination reveals marked fluctuance and induration under the erythematous region of the thigh. Which of the following is the best course of therapy? [image] A. incision and drainage with a linear incision B. CT scan of the thigh to rule-out necrotizing fasciitis C. antibiotics for one week, followed by reassessment D. incision and drainage, using a cruciate incision E. needle aspiration with a 30-gauge needle followed by antibiotics and reassessment within 5 days" - correct answer "A. incision and drainage with a linear incision The answer is A. A fluctuant, indurated area such as that pictured and described, tends to not respond to antibiotics (which cannot penetrate well into the abscess cavity). Cruciate incisions are unnecessary and risk wound healing problems. A 30-gauge needle is too small, and needle drainage of an abscess in this location is not generally used (it is more likely appropriate in facial abscesses).";"6What does the dotted line in the figure depict? [image] Figure used with permission from Hamilton et al, Emergency Medicine: An approach to clinical problem-solving A. Placement site for skin clamps. B. The needle entry angle that optimizes eversion of sutured skin edges. C. The approach for subcuticular suture. D. The injection plane for local anesthesia infiltration. E. Use of a finder needle"" to mark suture entry points.""" - correct answer "B. The needle entry angle that optimizes eversion of sutured skin edges. The answer is B. Eversion of the skin edges is maximized by directing the needle entry as shown in the figure. Injection for local anesthesia should usually be performed through the wound, SAEM EXAM QUESTIONS 2024-2025 ACTUAL EXAM REAL EXAM 1040 QUESTIONS AND CORRECT DETAILED ANSWERS WITH RATIONALES (VERIFIED ANSWERS) rather than through intact skin. Use of skin clamps can damage tissue; in cases where skin stabilization is needed gentle forceps application is preferred. Subcuticular sutures are placed deep to the skin.";"The components of the Figure (which is a photograph taken of the female perineal region) depict __________ (in the top of the Figure) which can be treated by placement of a __________ (in the lower part of the Figure): A. a cystocele -- pessary B. a benign tumor -- brachytherapy applicator C. a Bartholin's cyst -- Word catheter D. an inguinal lymph node -- gel-applicator for antibiotics administration E. a urinoma -- pediatric Foley catheter" - correct answer "C. a Bartholin's cyst -- Word catheter The answer is C. The patient's Bartholin's cyst will be drained, and placement of a Word catheter (inserted through an incision on the mucosal surface of the labia) will allow for continued drainage and healing.";"With respect to larygneal assessment, the Figure depicts what grading scale? [image] A. Macintosh B. Cormack-Lehane C. Mallampati D. Miller E. LMA" - correct answer "B. Cormack-Lehane The answer is B. The Cormack-Lehane scale allows communication of relative ease of visualization of laryngeal structures during laryngoscopy and intubation. The Miller and Macintosh are types of laryngoscope blades, and the LMA (laryngeal mask airway) is a type of airway.";"A 7 year old girl with severe asthma presents to the emergency department in severe respiratory distress. She clearly has difficulty breathing on her own and is obviously "tiring out." Her oxygen saturation is 85% and falling. The decision is made to intubate her. Of the following agents, which is often recommended (due to its bronchodilatory effects) as the induction agent of choice? SAEM EXAM QUESTIONS 2024-2025 ACTUAL EXAM REAL EXAM 1040 QUESTIONS AND CORRECT DETAILED ANSWERS WITH RATIONALES (VERIFIED ANSWERS) up to 75% of patients. --For further reading, see Rosen's Emergency Medicine: Concepts and Clinical Practices, 7th edition, pages 1391-2, 1396";"A patient presents with a question of foreign body in the foot. With reference to the figure, which of the following is true regarding anesthesia of the foot? [image] A. The patient is undergoing infusion of anesthesia into the saphenous vein. B. The patient shown is undergoing a sural nerve block. C. The patient shown is undergoing a posterior tibial nerve block. D. Local anesthesia (at the wound site) is preferred for wounds of the plantar foot. E. Adequate injection at the site depicted, will provide anesthesia for the entire sole of the foot." - correct answer "C. The patient shown is undergoing a posterior tibial nerve block. The answer is C. The nerve block depicted is a posterior tibial block. Regional blocks are preferred for procedures involving the plantar foot, since there is rich innervation and significant discomfort associated with injections into the soles. The posterior tibial nerve, located between the medial malleolus and Achilles tendon, supplies the medial portion of the sole and the medial side of the foot. The nerve runs next to the posterior tibial artery and is posterior to the pulse. In addition to the posterior tibial nerve block, a sural nerve block is frequently provided when anesthesia is desired for the heel and lateral foot.";"Which of the following is an absolute contraindication to surgical cricothyrotomy? A. Acute laryngeal disease B. Bleeding diathesis C. Age < 5 D. Massive neck edema" - correct answer "C. Age < 5 The answer is C. Given that surgical cricothyrotomy is often resorted to only after other techniques have been unsuccessful and/or the patient is not oxygenating or ventilating, most authors state that the only absolute contraindication is age. Because of the anatomic differences between children versus adults including the smaller cricothyroid membrane and the rostral funnel shaped more compliant pediatric larynx, surgical cricothyrotomy has been SAEM EXAM QUESTIONS 2024-2025 ACTUAL EXAM REAL EXAM 1040 QUESTIONS AND CORRECT DETAILED ANSWERS WITH RATIONALES (VERIFIED ANSWERS) contraindicated in infants and young children. However, the exact age at which a surgical cricothyrotomy can be done is controversial and not well defined. Various textbooks list the lower age limit from 5 years to 12 years. Choices A, C, and D are all relative contraindications to cricothyrotomy but may be overlooked in an emergent situation when the first priority is to obtain an airway.";"A 56 year old energy plant worker with a history of coronary heart disease and mild asthma severs a sharp, metal wire that snaps back and cuts his finger. He does not report a great deal of bleeding, but a 3-cm laceration on the distal right index finger requires sutures for repair. Of the approaches below, which is the best choice for pre-suturing anesthesia? A. lidocaine-epinephrine injection (lidocaine 2.0%, epinephrine 1:1000) as a digital nerve block anesthesia B. lidocaine-epinephrine injection (lidocaine 2.0%, epinephrine 1:1000) as a local infiltration anesthetic C. lidocaine-epinephrine topical solution (lidocaine 2.0%, epinephrine 1:1000) D. lidocaine injection (lidocaine 2%) as digital nerve block anesthetic" - correct answer "D. lidocaine injection (lidocaine 2%) as digital nerve block anesthetic The answer is D. In some cases surgical or dermatological consultants may utilize epinephrine- containing anesthetics in tissues with end-arterial blood supply. Although epinephrine- containing solutions are used routinely by podiatrists in digital blocks of the toes, without morbidity, when performing a digital block, it is advised to use anesthetics that do not contain epinephrine. If epinephrine-containing solutions are inadvertently used for a digital block in otherwise healthy individuals without peripheral vascular disease, it is unlikely that serious ischemic injury will occur. In the emergency department you should follow the general rule proscribing use of such agents in the digits, tip of the nose, penis, and pinna. The rationale for this prohibition is that vasoconstriction in these regions can result in ischemic complications, especially if the patient has underlying peripheral vascular disease. This patient was at risk. Standard concentrations of lidocaine are not likely to achieve effective analgesia when used topically.";"Regarding the laceration, in a 30-year old female, as depicted in the Figure, which of the following is true? [image] A. An infraorbital nerve block would provide adequate anesthesia to the area of the laceration B. If lidocaine anesthesia is to be used, the solution should not contain epinephrine SAEM EXAM QUESTIONS 2024-2025 ACTUAL EXAM REAL EXAM 1040 QUESTIONS AND CORRECT DETAILED ANSWERS WITH RATIONALES (VERIFIED ANSWERS) C. If there is not much tension on the wound, a topical adhesive (such as Dermabond) may be used to approximate the wound edges D. The eyebrow hair should be shaved to optimize the ability to closely approximate the wound edges E. This laceration is not suitable for topical anesthesia as an adjunct to repair" - correct answer "C. If there is not much tension on the wound, a topical adhesive (such as Dermabond) may be used to approximate the wound edges The answer is C. Eyebrow hair tends to not regrow after being shaved; a petroleum jelly can be used to "retract" the hair if necessary. If anesthesia is to be used to close this wound (wound glue would be a reasonable alternative), topical anesthesia is a good approach as long as care is taken to prevent drip into the eye. An infraorbital nerve block would not provide anesthesia to the area of the laceration. Epinephrine can be used safely in the supraorbital region.";"A patient sustains a forearm laceration as shown in the Figure. Regarding the wound, which of the following is true? [image] A. Due to presence of the flap, the wound should be closed with 6-0 suture. B. "Undermining" subcutaneous tissues may be a useful technique to reduce post-closure skin tension. C. Because of the high tension on the wound, a Penrose drain should be placed to reduce chances of infection. D. After shaving of the nearby hair, the wound is a good candidate for closure with tissue adhesives." - correct answer "B. "Undermining" subcutaneous tissues may be a useful technique to reduce post-closure skin tension. The answer is B. Undermining of skin "recruits" tissue by separating the skin from the deeper subcutaneous structures. Though in some cases, slight excision of protruding fatty tissue may be necessary, trimming of all visible fatty tissue is unnecessary. The wound is subject to high tension, and thus 6-0 suture or tissue adhesives are not optimal choices for closure. Drains are rarely indicated for initial wound care in the ED, and would not be necessary in the wound in the Figure.";"A 4-year old child sustained a large leg laceration while riding his bike. A medical student who was on his first clinical rotation was told to "numb up" the wound. The patient SAEM EXAM QUESTIONS 2024-2025 ACTUAL EXAM REAL EXAM 1040 QUESTIONS AND CORRECT DETAILED ANSWERS WITH RATIONALES (VERIFIED ANSWERS) The answer is A. This patient is exhibiting clinical signs of chest wall rigidity and glottic spasm, which is a rare but classic side effect of using high doses of intravenous fentanyl. Chest wall rigidity and glottic spasm, which may make ventilation difficult, are unique complications seen with very high doses of fentanyl given rapidly (generally > 15 mcg/kg). It has been observed at lower doses. This may not reliably be antagonized by naloxone and may require neuromuscular blockade and intubation to enable adequate ventilation. This complication is very rarely reported with the dosages of fentanyl used for PSA but can still happen.";"With respect to airway assessment, the Figure depicts what classification scale? [image shows tongues covering uvula to different degrees] A. LMA B. Mallampati C. Macintosh D. Cormack-Lehane E. Miller" - correct answer "B. Mallampati The answer is B. The Mallampati scale allows communication of ability to visualize structures of the posterior oropharynx, as a means of predicting ease of laryngoscopy and intubation. The Miller and Macintosh are types of laryngoscope blades, and the LMA (laryngeal mask airway) is a type of airway.";"A 46 year old male presents with acute onset frontal and bitemporal headache, associated with neck pain and sensitivity to bright lights. In the emergency department, the patient is febrile, reports several episodes of vomiting, and complains of worsening neck pain with head movement. A lumbar puncture is performed, the results of which are consistent with viral meningitis. A day later the patient complains of worsening headache. Which of the following is correct regarding this complication of lumbar puncture (LP)? A. A larger diameter needle decreases the incidence of post LP headache B. Lying supine for up to six hours will help prevent post LP headache C. Post LP headache is the most common complication of lumbar puncture D. Post LP headaches are typically unilateral and worse with supine position" - correct answer "C. Post LP headache is the most common complication of lumbar puncture SAEM EXAM QUESTIONS 2024-2025 ACTUAL EXAM REAL EXAM 1040 QUESTIONS AND CORRECT DETAILED ANSWERS WITH RATIONALES (VERIFIED ANSWERS) The answer is C. A smaller-diameter needle (not larger) is associated with a lower incidence of post-puncture headache because it causes a smaller dural hole (A). Simple analgesics are commonly prescribed, but they have no apparent advantage over bedrest and fluid intake. Lying supine for up to six hours carries no advantage over getting up after the procedure in the prevention of post spinal headache (B). Post LP headaches are typically bilateral and worse when sitting up (D). They improve with the supine position. Treatment of post-LP headache commonly involves keeping the patient supine to maximize intracranial CSF volume, use of oral caffeine, and for severe long-lasting headaches, autologous blood patch. The blood patch involves injecting one's own blood at the LP site in order to form a clot around the meningeal puncture site to avoid further leakage.";"All of the following may be indications for thoracentesis in the emergency department EXCEPT: A. evacuation of a simple stable pneumothorax (anterior approach) B. acute treatment of a large symptomatic pleural effusion (posterior approach) C. biopsy of a lung mass (anterior or posterior approach) D. diagnosis and treatment of suspected tension pneumothorax (anterior approach) E. diagnostic evaluation of a pleural effusion (posterior approach)" - correct answer "C. biopsy of a lung mass (anterior or posterior approach) The answer is C. All the other answers are common indications for performing a thoracentesis. Lung biopsy is not performed in the emergency department.";"A 21-year old male presents with a clean-knife wound sustained 24 hours ago. The laceration is 2cm in length, and located between the MCP and PIP levels of the nondominant hand index finger, on the flexor surface; the wound is well-approximated (i.e., not gaping). The patient has not had tetanus immunization within 10 years and has no complaints other than pain at the laceration site. Which of the following regarding this patient/presentation is TRUE? A. Since the wound appears superficial, there is no risk of involvement of the neurovascular bundle. B. It is too late (at 18 hours post-injury) to provide tetanus immunization. C. The wound should be sutured, primarily due to the potential for dehiscence due to tension on the skin edges. D. The wound should be sutured, primarily to minimize the chances of infection. SAEM EXAM QUESTIONS 2024-2025 ACTUAL EXAM REAL EXAM 1040 QUESTIONS AND CORRECT DETAILED ANSWERS WITH RATIONALES (VERIFIED ANSWERS) E. Simple wound cleaning, bandaging, and tetanus immunization are appropriate therapy for this pat - correct answer "E. Simple wound cleaning, bandaging, and tetanus immunization are appropriate therapy for this patient. The answer is E. Tetanus can be probably be effective if administered within the first few days of a wound; 24 hours is not too late. Suturing of this wound is not indicated, since it is an old wound, well-approximated, and is on the flexor surface of the digit where skin forces will be minimal. Though suturing is not indicated, careful assessment of the finger for neurovascular injury is appropriate given the anatomical location of the wound over the bundle (the superficial appearance of the wound may be misleading).";"A 25 year old male college student with a history of type I diabetes mellitus presents to the emergency department complaining of worsening headache, vomiting, and fever. On exam, he has a temperature of 101.6 F, meningismus, and photophobia. A lumbar puncture is performed and reveals a CSF glucose consistent with bacterial meningitis. What is the normal ratio between CSF and serum glucose? A. 1:1 B. 2:1 C. 0.4:1 D. 0.6:1" - correct answer "D. 0.6:1 The answer is D. The normal range of CSF glucose is 50 to 80 mg/dL, which is 60% to 70% of the glucose concentration in the blood. Ventricular fluid glucose levels are 6 to 8 mg/dL higher than in lumbar fluid. A ratio of CSF glucose-to-blood glucose of less than 0.5 or a CSF glucose level below 40 mg/dL is invariably abnormal. The ratio is higher in infants, for whom a ratio of less than 0.6 is considered abnormal. Hyperglycemia may mask a depressed CSF glucose level; when present, the CSF glucose-to-blood glucose ratio should be measured routinely. With extreme hyperglycemia, a ratio of 0.3 is abnormal. In patients with systemic hyperglycemia, the ratio changes to 0.4:1.";"An 18 year old male is transported to the emergency department after being involved in a motor vehicle collision. On initial evaluation, he is found to be comatose, hypotensive, and is diagnosed clinically as having a tension pneumothorax on the left side. What is the correct statement regarding needle decompression or chest tube placement in this patient? A. Placement of the needle should be in the 3rd ICS, midaxillary line. B. An 18 French chest tube would be appropriate in this situation. SAEM EXAM QUESTIONS 2024-2025 ACTUAL EXAM REAL EXAM 1040 QUESTIONS AND CORRECT DETAILED ANSWERS WITH RATIONALES (VERIFIED ANSWERS) C. Heroin D. Insulin" - correct answer "B. Cocaine Cocaine is a sympathomimetic. Sympathomimetics and anticholinergics such as Jimson Weed can be differentiated by the presence of sweating although both can cause delirium and mydriasis. Aspirin or salicyclate toxicity can cause increased respiratory drive through direct stimulation of the medullary respiratory center but should not cause papillary changes. Heroin will result in a classic toxidrome of miosis, CNS and respiratory depression as will other opiates.";"A mother brings in her 4 year old child who was happily eating "blackberries" from weeds in the garden and is now acting strangely. She has identified them as Belladonna from a quick internet search. Which physical examination finding might you also expect to find in this child? A. Urinary incontinence B. Miosis C. Flushed skin D. Diaphoresis" - correct answer "C. Flushed skin The answer is C. The classic presentation of anticholinergic toxicity is best remembered by the following: hot as Hades; blind as a bat; dry as a bone; red as a beet; mad as a hatter. Patients with anticholinergic toxicity are flushed, warm, psychotic, mydriatic, and dry. Bowel sounds are classically hypoactive.";"With which of the following substances is acute withdrawal most likely life-threatening? A. Lithium B. Cocaine C. Ethanol D. Heroin" - correct answer "C. Ethanol";"For which of the following cases is activated charcoal therapy most appropriate? A. Drain cleaner ingestion B. Iron supplement overdose C. Lithium overdose SAEM EXAM QUESTIONS 2024-2025 ACTUAL EXAM REAL EXAM 1040 QUESTIONS AND CORRECT DETAILED ANSWERS WITH RATIONALES (VERIFIED ANSWERS) D. Acetaminophen overdose" - correct answer "D. Acetaminophen overdose Charcoal acts by adhering to most toxins, impairing toxin absorption, and enhancing elimination. Some toxins (i.e., heavy metals such as lithium, lead, and iron) do not bind to charcoal. Consequently, charcoal is not indicated in isolated heavy metal injections. Also, charcoal is contraindicated in patients with unprotected airways (risk of aspiration) and in caustic ingestions as the black color of the charcoal interferes with the endoscopic evaluation that often follows caustic ingestion. In addition, a caustic ingestion such as alkaline drain cleaner causes its damage by direct contact rather than absorption so charcoal will not be effective.";"A young woman presents with an amitriptyline overdose. She is agitated and confused. In overdoses of this class of medications, an indicator of severe toxicity would include A. Serum amitriptyline level > 200 mcg/dl B. Prolonged QRS interval C. Metabolic acidosis with a pH < 7.25 D. Elevated osmolar gap > 20" - correct answer "B. Prolonged QRS interval";"A 17 year-old has presented after taking a large amount of nortriptyline prescribed for migraine prophylaxis. Clinically, you take care of stabilizing her and initiate appropriate treatment. After reviewing reference materials you calculate that she has taken a potentially lethal dose of this tricyclic antidepressant. Which of the following would you expect to see on her electrocardiogram? A. Prolonged PR intervals B. Prolonged QRS intervals C. Right bundle branch block D. Compacted QT intervals" - correct answer "B. Prolonged QRS intervals The answer is B. Tricyclic antidepressant (TCA) toxicity can result in the following EKG abnormalities: sinus tachycardia (through antimuscarinic activity), prolongation of any of the EKG intervals (through sodium and potassium channel blockade), ventricular dysrhythmias (sodium channel blockade), and right axis deviation of the terminal 40 ms of the QRS complex SAEM EXAM QUESTIONS 2024-2025 ACTUAL EXAM REAL EXAM 1040 QUESTIONS AND CORRECT DETAILED ANSWERS WITH RATIONALES (VERIFIED ANSWERS) (sodium channel blockade).";"A 45 year-old is brought in 8 hours after a large overdose of his lithium. What is the best treatment method for this overdose? A. Whole bowel irrigation B. Activated charcoal C. Hemodialysis D. Gastric lavage" - correct answer "C. Hemodialysis The answer is C. Dehydration, over-diuresis, and drug-drug interaction (particularly NSAIDs) are common precipitants of lithium toxicity in the patient chronically taking lithium. In general, the clinical condition, not drug level, should guide therapy. In acute ingestions in particular, lithium levels do not correlate well with symptoms or prognosis. Charcoal does not bind heavy metals like lithium. Hemodialysis is helpful in lithium toxicity.";"A 42 year-old woman presents with an overdose of her Xanax (alprazolam) that her family indicates she has been taking for years to help with her anxiety. The bottle indicates that the prescription was filled yesterday with 90 pills and is now empty. The patient is minimally responsive to painful stimuli and does not react when you suction secretions out of her posterior pharynx. What is your next management step? A. Administration of narcan B. Close observation C. Intubation for airway support D. Administration of flumazenil" - correct answer "C. Intubation for airway support The answer is C. Isolated benzodiazepine (BZD) OD is generally quite benign. When taken in combination with other agents, however, BZDs can cause significant morbidity and mortality. Patients with BZD OD commonly present with oversedation. A paradoxical excitation syndrome can occur but is uncommon. While an antidote (flumazenil) exists, supportive care is the key to treatment. Flumazenil, a BZD antagonist, can cause seizures in patients taking BZDs chronically by inducing an acute BZD withdrawal syndrome. It is best used in reversal of BZD-induced iatrogenic oversedation. Here, however, physicians must take heed as the half-life is short and resedation can occur.";"A 26 year-old presents with agitation, chest pain and a heart rate of 142 bpm after intranasal cocaine use. The EKG is normal except for sinus tachycardia. What is the best medication to use in this situation? SAEM EXAM QUESTIONS 2024-2025 ACTUAL EXAM REAL EXAM 1040 QUESTIONS AND CORRECT DETAILED ANSWERS WITH RATIONALES (VERIFIED ANSWERS) by his family for vomiting and diarrhea. He also complains that things "have weird colors". He has been having odd palpitations but cannot describe them further. His family expresses their concern that he has not been taking his medications correctly. Given his presenting symptoms, which medication are you most concerned about? A. Amiodarone B. Diphenhydramine C. Metoprolol D. Digoxin" - correct answer "D. Digoxin The answer is D. Digoxin toxicity classically presents as weakness, fatigue, nausea/vomiting/diarrhea, confusion, and a visual disturbance hallmarked by yellow/green halos around objects.";"The clinical presentation of clonidine toxicity most closely mimics toxicity from which of the following classes of medication? A. Opioids B. Beta blockers C. Cholinergics D. Stimulants" - correct answer "A. Opioids The answer is A. The hallmark signs and symptoms of clonidine toxicity include:hypotension, bradycardia, mental status change, respiratory depression, and miosis. The presentation very closely mimics opioid toxicity.";"A patient presents after an unknown ingestion. Her initial electrocardiogram (EKG) is shown below [image shows sinus tachy] Based on the EKG, an overdose with which of the following medications would be most likely? A. Amitriptyline B. Ibuprofen C. Nifedipine SAEM EXAM QUESTIONS 2024-2025 ACTUAL EXAM REAL EXAM 1040 QUESTIONS AND CORRECT DETAILED ANSWERS WITH RATIONALES (VERIFIED ANSWERS) D. Clonidine" - correct answer "A. Amitriptyline The answer is A. Amitriptyline is a tricyclic antidepressant (TCA). As such, it has anticholinergic activity that will cause a sinus tachycardia. Additional EKG findings with TCA toxicity include interval prolongation and terminal 40 ms right axis deviation. Clonidine, nifedipine, and metoprolol typically cause bradycardia. NSAIDs, like ibuprofen, rarely affect the heart rate.";"A 2 year old child presents with an overdose of her mother's iron containing multivitamins. What antidote should you consider for iron toxicity? A. Pyridoxine B. Glucagon C. Deferoxamine D. Methylene blue" - correct answer "C. Deferoxamine The answer is C. Deferoxamine binds directly to free iron and thus is the antidote for iron toxicity. It is given intramuscularly or intravenously and often causes the patient's urine to turn color (vin rosé urine). Methylene blue is an antidote for methemoglobinemia. N-acetylcysteine is the antidote for acetaminophen. Pyridoxine is the antidote for isoniazid toxicity and glucagon can serve as an antidote for beta blocker, calcium channel blocker, or insulin overdoses.";"A 3 year old girl presents after accidentally ingesting an alkali drain cleaner. Which of the following statements regarding her management is true? A. Neutralization therapy using a strong acid is warranted B. Gastric lavage should be performed immediately to reduce gastric injury C. Endoscopy is useful in the assessment of injury D. Activated charcoal should be administered" - correct answer "C. Endoscopy is useful in the assessment of injury The answer is C. Gastric lavage and ipecac therapy are contraindicated due to concern regarding recurrent injury to the esophagus from a second contact with the caustic ingestant. Activated charcoal is contraindicated because it obscures endoscopic assessment (and doesn't work at binding caustics). Neutralization with milk or water may be indicated in caustic ingestions without perforation, but strong acids/alkali should not be used. Endoscopic SAEM EXAM QUESTIONS 2024-2025 ACTUAL EXAM REAL EXAM 1040 QUESTIONS AND CORRECT DETAILED ANSWERS WITH RATIONALES (VERIFIED ANSWERS) assessment should be performed early as the risk of procedurally-induced perforation increases with delayed endoscopy.";"Severe lead toxicity can commonly result in which of the following clinical symptoms A. Stocking glove peripheral neuropathy B. Constipation C. Dermatitis D. Memory loss" - correct answer "D. Memory loss The answer is D. Lead toxicity affects a variety of systems. The central nervous system effects are many and range from encephalopathy and seizure to sleep disturbance and memory deficits. The peripheral nervous system can also be involved, with paresthesias and wrist drop being common. Colicky abdominal pain is often present. While dermatitis is not common in lead poisoning, you can see bluish lead lines on the gingiva.";"A 55 year-old man is brought down from the outpatient procedures clinic after becoming severely short of breath during an endoscopy under light sedation. His pulse oximeter is reading 100% on a non-rebreather mask. You notice an interesting discoloration of his blood when it is drawn. What antidote should be administered? A. Deferoxamine B. Methylene blue C. Hydroxycobalamin D. Amyl nitrite" - correct answer "B. Methylene blue The answer is B. Nitrates/nitrites, local anesthetics, dapsone, and phenazopyridine are the common causes of methemoglobinemia. Methemoglobinemia causes the oxygen dissociation curve to shift to the left, making the remaining hemoglobin less likely to give up oxygen to the tissues. Blood from patients with methemoglobinemia is a chocolate brown color. Methylene blue is the antidote. Pulse oximetry is unreliable in patients with methemoglobinemia, since the pulse oximeter cannot differentiate oxyhemoglobin from methemoglobinemia.";"A 27 year old woman is brought into the emergency department by her roommate 30 minutes after ingesting a bottle of aspirin in a suicide attempt. Which of the following acid-base disorders is most likely to be present in this patient? A. Primary metabolic acidosis with compensatory respiratory alkalosis SAEM EXAM QUESTIONS 2024-2025 ACTUAL EXAM REAL EXAM 1040 QUESTIONS AND CORRECT DETAILED ANSWERS WITH RATIONALES (VERIFIED ANSWERS) C. A chest radiograph that was taken with the patient rotated D. Right upper lobe pneumonia" - correct answer "A. Mediastinal shift due to fluid in the right hemithorax The answer is A. The patient is not intubated. The pathology in the right hemithorax appears as hyperdensity, rather than air density (not a pneumothorax), and involves more than the right upper lobe.";"Regarding the epidemiology of asthma in the United States, which of the following is true? A. Incidence is comparable for Caucasians and African-Americans B. Etiology is thought to be genetic, not environmental C. Prevalence increased in the 1980's, and then decreased in the 1990's D. More common in males than females in adult and pediatric populations" - correct answer "C. Prevalence increased in the 1980's, and then decreased in the 1990's The answer is C. Despite an increase in asthma prevalence in the United States, Canada, Great Britain and Australia in the 1980s, the 1990s saw a decrease in prevalence in these areas. Regarding gender, male children are more likely than female children to have asthma, however the reverse is true with adults. African-Americans have a higher prevalence of asthma than Caucasians. Migrants who relocate from an area of low asthma prevalence to an area of high asthma prevalence tend to have an increased prevalence of asthma suggesting a role for environmental factors in the development of asthma.";"Regarding pulsus paradoxus and asthma, which of the following statement s is correct A. Pulsus paradoxus is pathognomonic for asthma B. The absence of pulsus paradoxis in asthma rules out severe disease C. The presence of pulsus paradoxis in asthma indicates severe disease D. Pulsus paradoxus is a fall in systolic blood pressure during inspiration" - correct answer "C. The presence of pulsus paradoxis in asthma indicates severe disease SAEM EXAM QUESTIONS 2024-2025 ACTUAL EXAM REAL EXAM 1040 QUESTIONS AND CORRECT DETAILED ANSWERS WITH RATIONALES (VERIFIED ANSWERS) The answer is C. Pulsus paradoxus is defined as a fall in systolic blood pressure of greater than 10mm Hg upon inspiration. It is typically present during acute asthma exacerbations in severe asthma; however, its absence does not rule out severe disease. Although initially present, a pulsus paradoxus may disappear after only minimal improvement in air flow through the larger airways. Pulsus parodoxus may occur in other diseases besides asthma (for example, pericardial tamponade).";"Which of the following is correct regarding the use of corticosteroids in acute asthma exacerbation? A. Beneficial effects occur within the first hour of administration. B. Intravenous steroids are superior to the oral route C. Tapering is needed with all corticosteroid regimens D. Inhaled steroids should be avoided" - correct answer "D. Inhaled steroids should be avoided The answer is D. Oral and intravenous steroids are equally efficacious in treating an asthma exacerbation. Yet, in the setting of a severe asthma exacerbation, a patient may have difficulty taking oral medications and the intravenous route is preferred.";"Which of the following is true regarding the treatment of acute asthma exacerbation in the Emergency Department? A. Anticholinergics by inhalation may be beneficial B. Intravenous albuterol may be indicated C. Heliox should only be used in the intubated patient. D. Intramuscular terbutaline is preferred over intravenous" - correct answer "A. Anticholinergics by inhalation may be beneficial The answer is A. Salmeterol is a long-acting beta2-selective adrenergic agonist that has no role in the treatment of an acute asthma exacerbation, but it is frequently preferred for outpatient asthma management due to its BID dosing schedule.";"Which of the following drugs is MOST beneficial in an acute COPD exacerbation? A. Beta adrenergic agonists such as albuterol B. Mucokinetic agents such as acetylcysteine C. Steroids such as solumedrol SAEM EXAM QUESTIONS 2024-2025 ACTUAL EXAM REAL EXAM 1040 QUESTIONS AND CORRECT DETAILED ANSWERS WITH RATIONALES (VERIFIED ANSWERS) D. Methylxanthines such as theophylline" - correct answer "A. Beta adrenergic agonists such as albuterol The answer is A. Mucokinetic agents should not be used acutely in treatment of COPD exacerbation. These agents act to mobilize secretions, and this increases the work of the patient's breathing.";"Regarding the pathogens involved in community-acquired pneumonia, which of the following is true? A. Co-infection with multiple bacteria, such as Chlamydia and Strep pneumoniae commonly occur B. Milder cases of community acquired pneumonia are frequently caused by Chlamydia C. Q fever, caused by Coxiella burnetii, may present as pneumonia, particularly in patients exposed to rabbits D. Etiologic agents for patients admitted to the ICU with pneumonia most commonly include Neisseria meningitidis and Strep pneumoniae" - correct answer "A. Co-infection with multiple bacteria, such as Chlamydia and Strep pneumoniae commonly occur The answer is A. Co-infection with multiple bacteria, such as Chlamydia and S. pneumoniae, is a well-recognized occurrence and should be sought out to ensure appropriate antibiotic coverage.";"The next step in treatment for a patient with ventricular fibrillation, which is refractory to multiple countershocks and epinephrine, is: A. calcium B. transcutaneous pacing C. adenosine D. lidocaine E. bicarbonate" - correct answer "D. lidocaine OUTDATED SAEM EXAM QUESTIONS 2024-2025 ACTUAL EXAM REAL EXAM 1040 QUESTIONS AND CORRECT DETAILED ANSWERS WITH RATIONALES (VERIFIED ANSWERS) solution every 3 to 5 minutes. Doses >1 milligram are not recommended and may be harmful."";"A patient with the rhythm shown in the Figure should be treated with: [image shows asystole] A. amiodarone B. verapamil C. defibrillationdefibrillation D. epinephrine" - correct answer "D. epinephrine The answer is D. The flat line above is characteristic of asystole. This is mechanical and electrical standstill. It is important to check that the monitor is working and to check a second lead to rule out very fine ventricular fibrillation. The treatment of choice is epinephrine or vasopressin and atropine";"A 75 year old male is brought in by EMS with CPR in progress. He has been shocked three times and received 1 mg of epinephrine. His rhythm strip as shown in the Figure reveals: [image random up and down strokes] A. atrial fibrillation B. atrial flutter C. ventricular fibrillation D. ventricular tachycardia" - correct answer "C. ventricular fibrillation The answer is C. Ventricular fibrillation is the totally disorganized depolarization and contraction of the ventricles. The EKG is characterized by a variable zig-zag pattern of very rapid, chaotic, and grossly irregular deflections of irregular appearance and varying amplitude. There are no discernible P waves, ST segments, T waves, or even QRS complexes.";"A 50 year old male presents to the emergency department 1 week after having an acute myocardial infarction. He now complains of dizziness. His EKG, shown below, is characterized by: [image all over the place] SAEM EXAM QUESTIONS 2024-2025 ACTUAL EXAM REAL EXAM 1040 QUESTIONS AND CORRECT DETAILED ANSWERS WITH RATIONALES (VERIFIED ANSWERS) A. normal sinus rhythm B. complete heart block C. second degree AV block Mobitz Type 2 D. second degree AV block Mobitz Type 1" - correct answer "B. complete heart block The answer is B. In complete AV block, there is no relationship between the P waves (atrial beats) and the ventricular beats. The latter arises from different foci, thus the QRS complex is wide (impulse is not conducted through the normal pathways) and the rate is often slow at < 50/minutes. Complete heart block is common in the setting of ischemia and also in the peri- infarct period. The treatment for complete AV block in this setting is usually permanent pacing.";Which of the statements below is correct regarding the two airway maneuvers depicted in the figure's left (labeled A") and right (labeled "B") aspects? - correct answer ;[image A:air into mouth B: chin lift jaw thrust] - correct answer ;Figure used with permission from Hamilton et al, Emergency Medicine: An approach to clinical problem-solving - correct answer ; - correct answer A.; - correct answer B.; - correct answer C.; - correct answer D.; - correct answer E.;"With respect to laboratory findings in diabetic ketoacidosis (DKA) and hyperglycemic hyperosmolar nonketotic coma (HHNC), all of the following guidelines are generally true EXCEPT: A. Serum bicarbonate is typically severely low (<10mEq) in patients with either DKA or HHNC. B. Serum osmolality in patients with HHNC is typically > 350 mOsm/L. C. BUN is elevated more in patients with HHNC (>50 mg/dL) than in patients with DKA (25-50 mg/dL). D. Patients with HHNC typically have blood glucose > 700 mg/dL, whereas patients with DKA have blood glucose > 350 mg/dL. E. Serum ketones are present in patients with DKA but not usually in patients with HHNC." - correct answer "A. Serum bicarbonate is typically severely low (<10mEq) in patients with either DKA or HHNC. The answer is A. Although patients with DKA typically have profound metabolic acidosis with serum bicarbonate < 10mEq, acidosis is typically absent in patients with HHNC and serum bicarbonate is usually > 15 mEq.";"In a 70kg male DKA patient with serum glucose of 573 SAEM EXAM QUESTIONS 2024-2025 ACTUAL EXAM REAL EXAM 1040 QUESTIONS AND CORRECT DETAILED ANSWERS WITH RATIONALES (VERIFIED ANSWERS) mg/dL, all of the following statements with regard to fluid and electrolyte imbalances are true EXCEPT: A. A normal magnesium level is reassuring and obviates the need for magnesium replacement. B. The patient is likely to be total body phosphorus depleted. C. Total body water deficit is approximately 5L. D. Serum sodium of 129 mEq represents dilutional hyponatremia and the corrected value is approximately 137 mEq. E. Despite a serum potassium level of 4.8 mEq, the patient is probably total body potassium depleted." - correct answer "A. A normal magnesium level is reassuring and obviates the need for magnesium replacement. The answer is A. Patients with DKA are typically severely dehydrated with a total body water deficit of approximately 70-80 mL/kg, in addition to being total body depleted of potassium, magnesium, and phosphorous despite initially normal serum levels of these electrolytes.";"Regarding the treatment of hyperosmolar hypertonic nonketotic coma (HHNC) and its associated symptoms, which of the following is correct: A. Hyperosmolarity should be corrected within the first few hours in the emergency department. B. Since patients are not acidotic, close monitoring of glucose is not necessary. C. In HHNC patients with severe dehydration, bleeding diathesis is a major clinical concern. D. Half of the fluid deficit should be corrected over the first hour and the remainder over the following 8 hours. E. Phenytoin (Dilantin) is often ineffective for seizures associated with HHNC." - correct answer "E. Phenytoin (Dilantin) is often ineffective for seizures associated with HHNC. The answer is E. Phenytoin (Dilantin) is contraindicated in patients with HHNC as it may impair endogenous insulin release and is often ineffective in the management of seizures associated with HHNC. Half of the fluid deficit should be replaced over the first 8 hours, and the remainder over the ensuing 24 hours. Glucose must be tightly monitored as fluid resuscitation alone may normalize serum glucose or precipitate hypoglycemia in aggressive fluid resuscitation. Too- SAEM EXAM QUESTIONS 2024-2025 ACTUAL EXAM REAL EXAM 1040 QUESTIONS AND CORRECT DETAILED ANSWERS WITH RATIONALES (VERIFIED ANSWERS) The answer is D. Glucagon is ineffective in patients without adequate glycogen stores, as would be expected in alcoholics. Further, glucagon can precipitate a severe lactic acidosis in patients with glycogen storage diseases and therefore should not be used in children with hypoglycemia of unknown etiology. Typical symptoms of hypoglycemia include sweating, tachycardia, nervousness, hunger, and neurologic symptoms. Symptoms should not be attributed to hypoglycemia unless the level falls below 40-50 mg/dL. Type 1 diabetics practicing strict control of serum glucose are at high risk for hypoglycemic episodes precipitated by skipping a meal, or by increasing energy output or insulin dose. Due to the extended half-lives of the oral hypoglycemic agents, hospitalization and 24-hour observation (at minimum) are the typical management for overdose of these agents.";"A 20 year old man was ice skating on a frozen pond and fell through the ice. The water was only about six feet deep and he was able to keep his head above water while bystanders were able to extract him after 10 minutes. The patient was transported to the emergency department and had an initial core temperature of 30 C. The patient's wet clothes were immediately removed and rewarming was initiated. Which of the following physical examination findings is expected? A. Tachycardia B. Hyperventilation C. Altered mental status D. shivering" - correct answer "C. Altered mental status The answer is C. Moderate hypothermia is associated with temperatures of 28-32 C. Shivering ceases at about 32 degrees Celsius. Moderate hypothermia is associated with altered mental status, absence of shivering, bradycardia, and bradypnea";"A 36 year old male backpacking in the wilderness loses his way in a snowstorm. Temperatures are well below zero degrees and his clothing is inadequate. He is rescued 5 days later and presents to the ED. Rescue crew has already initiated passive rewarming and have removed patient's damp clothing. On arrival, vital signs show pulse of 100 and temperature of 35.5C. On physical exam, you note patient has several toes that are purple with hemorrhagic blisters on his feet. Which of the following is the most appropriate initial management? A. Tetanus prophylaxis B. Administration of morphine C. Immersion in warm water bath SAEM EXAM QUESTIONS 2024-2025 ACTUAL EXAM REAL EXAM 1040 QUESTIONS AND CORRECT DETAILED ANSWERS WITH RATIONALES (VERIFIED ANSWERS) D. Debridement of necrotic tissue" - correct answer "C. Immersion in warm water bath The answer is C. Immersion of the affected extremity is the mainstay of treatment for patients with frostbite. Numbness of the affected area is the most common initial symptom and severe pain is frequently encountered after rewarming. Tetanus prophylaxis and debridement is indicated , but is not the most appropriate initial step in the management of patients with frostbite.";"All of the following are true regarding the epidemiology of hypothyroidism EXCEPT: A. Hypothyroidism does not occur in infants under six months of age. B. Most cases of hypothyroidism manifest in the winter months. C. Approximately half of myxedema cases are diagnosed after admission to the hospital. D. Hypothyroidism occurs three to ten times more frequently in women than men. E. Peak incidence of hypothyroidism is in the seventh decade." - correct answer "A. Hypothyroidism does not occur in infants under six months of age. The answer is A. Hypothyroidism may occur at any age including the very young, but is infrequently seen in infants due to regular newborn screening for hypothyroidism. The increased frequency of the disease in women is attributed to the increased prevalence of autoimmune thyroid conditions in women. The majority of cases present in winter months due to the body's decreased ability to accommodate to cold weather in a hypothyroid state.";"With regard to laboratory findings in hypothyroidism, which of the following is false? A. Total thyroxine levels may be normal due to elevated thyroxine-binding globulin (TBG) levels. B. Free T4 and TSH levels are typically low in secondary and tertiary hypothyroidism. C. Serum thyroid-stimulating hormone (TSH) is the most sensitive test to diagnose primary hypothyroidism. D. T3 level may be normal in hypothyroid states. E. Free thyroxine (T4) is always depressed in hypothyroid states." - correct answer "E. Free thyroxine (T4) is always depressed in hypothyroid states. SAEM EXAM QUESTIONS 2024-2025 ACTUAL EXAM REAL EXAM 1040 QUESTIONS AND CORRECT DETAILED ANSWERS WITH RATIONALES (VERIFIED ANSWERS) The answer is E. Free T4 may be normal in early stages of hypothyroidism due to physiologic compensation from elevated TSH levels.";"Metabolic abnormalities often seen with hypothyroidism include all of thefollowing EXCEPT: A. hyperglycemia B. respiratory acidosis from hypoventilation C. anemia D. hyponatremia E. hypercholesterolemia" - correct answer "A. hyperglycemia The answer is A. Hyperglycemia is not typically associated with hypothyroidism. Hypoglycemia may be present, but is unusual, and may suggest hypothalamic-pituitary involvement. Hyponatremia is common and corrects with thyroid replacement. Hypercholesterolemia to over 250 mg/dL is typical. A mild normochromic, normocytic anemia may be present, in addition to respiratory acidosis from hypoventilation.";"Laboratory abnormalities typically seen with adrenal insufficiency include all of the following EXCEPT: A. hypercalcemia B. azotemia C. hypokalemia D. hyponatremia E. hypoglycemia" - correct answer "C. hypokalemia The answer is C. Hyperkalemia is seen in approximately 64% of patients with adrenal failure. Typically this is because of aldosterone production failure that normally enhances potassium excretion. Even more common is hyponatremia, present in 88% of patients. Hypoglycemia is present in two-thirds of patients and is a significant cause of morbidity and mortality associated with adrenal failure. Hypercalcemia is seen in 6 to 33% for unclear reasons; azotemia and increased hematocrit from hypovolemia may also be present.";"Regarding the treatment of suspected but not confirmed adrenal insufficiency, which of the following is most appropriate? A. hydrocortisone 100mg IV every 6 hours B. cortisone 100mg IM every 6 hours SAEM EXAM QUESTIONS 2024-2025 ACTUAL EXAM REAL EXAM 1040 QUESTIONS AND CORRECT DETAILED ANSWERS WITH RATIONALES (VERIFIED ANSWERS) The answer is C. The syndrome of high altitude illness ranges from mild AMS (Acute Mountain Sickness) to life threatening conditions of HAPE (High Altitude Pulmonary Edema) and HACE (High Altitude Cerebral Edema). This student is experiencing mild AMS. After the symptoms of altitude illness occur, further ascent to a higher sleeping altitude is contraindicated. Halting ascent or activity to allow further acclimatization may reverse symptoms. Acetazolamide is a carbonic anhhydrase inhibitor that induces a renal bicarbonate diuresis, causing a metabolic acidosis and thereby increasing ventilation and arterial oxygenation. Supplemental oxygen addresses the hypoxic insult of high altitude exposure. Ibuprofen is useful for the treatment of his headache. Dexamethasone can help with the symptoms of AMS, but does not play a role in acclimatization.";"A 65-year-old female presents with a chief complaint of palpitations and dyspnea on exertion. Vital signs are BP 130/84, HR 160 (and irregularly irregular), RR 14, T 37.8. EKG shows a narrow complex, irregularly irregular rhythm with absence of p-waves and an undulating baseline. What endocrine abnormality is most likely to be a direct cause of this abnormal rhythm? A. Cushing's syndrome B. Hyperthyroidism C. Hyperparathryoidism D. Addison's disease" - correct answer "B. Hyperthyroidism The answer is B. Atrial fibrillation is a common arrhythmia. Its hallmark is the absence of P waves and irregular rhythm. It is associated with many medical conditions including ischemic heart disease and thyrotoxicosis. Atrial fibrillation increases the risk of thrombus formation and arterial embolism. AF's many treatment options include calcium channel blockers, beta blockers, amiodarone, quinidine, and cardioversion. Pacing is not a treatment option.";"Regarding the diagnosis and treatment of thyroid storm in the emergency department, which of the following is true? A. Patients suspected of having thyroid storm should undergo treatment prior to a definitive diagnosis due to the potentially life-threatening nature of this disease. B. Thyroid storm cannot be diagnosed in the absence of altered mental status. C. The diagnosis of thyroid storm is generally a straightforward clinical diagnosis and rarely confused clinically with other disorders such as psychiatric or other endocrine disorders. D. A stat thyroid-stimulating hormone (TSH) level is required to make the diagnosis. SAEM EXAM QUESTIONS 2024-2025 ACTUAL EXAM REAL EXAM 1040 QUESTIONS AND CORRECT DETAILED ANSWERS WITH RATIONALES (VERIFIED ANSWERS) E. Treatment of thyroid storm should only be undertaken after consultation with an endocrinologist." - correct answer "A. Patients suspected of having thyroid storm should undergo treatment prior to a definitive diagnosis due to the potentially life-threatening nature of this disease. The answer is A. The diagnosis of thyroid storm in the emergency department may be challenging due to the relatively infrequent occurrence of the disease and its typically nonspecific signs and symptoms. Treatment should be initiated in a timely fashion in any patient suspected of having thyroid storm due to the potential lethality of this disease. Immediate laboratory testing is typically not available to confirm clinically suspected cases, although thyroxine (T4) radioimmunoassay and free T4 index are good screening tests for hyperthyroidism. Clinical presentation of thyroid storm may be mistaken for psychiatric illness, heat stroke, sympathomimetic toxidromes, hypoglycemia and withdrawal syndromes, among others. Altered mental status, though frequently present, is not a prerequisite for diagnosis.";"Which of the following is not a common sign or symptom of thyrotoxicosis? A. hyperhidrosis B. nervousness C. tachycardia D. congestive heart failure E. hypothermia" - correct answer "E. hypothermia The answer is E. Fever, not hypothermia, is commonly seen in thyrotoxicosis. Other common signs and symptoms include tachycardia, congestive heart failure, wide pulse pressure, tremor, thyrotoxic stare, thyromegaly, nervousness, weight loss, and palpitations.";"Which of the following is true regarding the use of iodine in the treatment of thyroid storm? A. Iodine should be administered at least one hour after propylthiouracil (PTU) has been given. B. Dexamethasone must be given 30 minutes prior to iodine administration. C. Iodine should be administered even in patients with known iodine allergy. D. Iodine should be the first drug administered in the treatment of thyroid storm. E. Iodine should be administered only after treatment with propranolol." - correct answer "A. Iodine should be administered at least one hour after propylthiouracil (PTU) has been given. SAEM EXAM QUESTIONS 2024-2025 ACTUAL EXAM REAL EXAM 1040 QUESTIONS AND CORRECT DETAILED ANSWERS WITH RATIONALES (VERIFIED ANSWERS) The answer is A. Iodine inhibits preformed thyroid hormone release and should be administered at least one hour after treatment with PTU to prevent organification of iodine. A typical dose is potassium iodide (SSKI) 5 drops every 6 hours PO or NG, or sodium iodide 1 gm slow IV drip every 8 to 12 hours. Iodine should not be administered to patients with known iodine allergy.";"All of the following are commonly used in the supportive treatment of thyroid storm EXCEPT: A. corticosteriods B. oxygen C. acetaminophen to manage hyperpyrexia D. amiodarone to control dysrhythmias E. diuretics to treat congestive heart failure" - correct answer "D. amiodarone to control dysrhythmias The answer is D. Amiodarone is an iodine-rich antidysrhythmic with poorly-defined effects on thyroid function that has been associated with both hyperthyroidism and hypothyroidism. It should therefore be avoided in the management of thyroid disease. Propranolol is standard therapy in thyroid storm and, in addition to its effects of adrenergic blockade, also may reduce dysrhythmias. Of note, aspirin should be avoided in the treatment of hyperpyrexia as it may increase the level of active thyroid hormone by displacing thyroid hormone from thyroglobulin.";"A 4 year old girl is brought to the ED two hours after being stung by a scorpion while on a camping trip in Arizona. She has periods of agitation and restlessness alternating with calmness. Her vital signs are: blood pressure 106/61, pulse 120, respiratory rate 24, temperature 37.0C, and oxygen saturations of 99% on room air. On physical examination you note drooling, a disconjugate gaze, and occasional jerking movements of the extremities. Which of the following is the most correct regarding the treatment of a scorpion sting in this child? A. Complications of treatment with antivenom include delayed serum sickness B. Treatment with antivemon is not indicated because these symptoms will be self-limiting C. The patient should be intubated because respiratory failure is expected D. Analgesics have a minimal role in controlling symptoms" - correct answer "A. Complications of treatment with antivenom include delayed serum sickness SAEM EXAM QUESTIONS 2024-2025 ACTUAL EXAM REAL EXAM 1040 QUESTIONS AND CORRECT DETAILED ANSWERS WITH RATIONALES (VERIFIED ANSWERS) B. Apply a Bair Hugger C. Administer warm IV fluids D. Immerse in a warm water bath at 40 Celcius" - correct answer "C. Administer warm IV fluids The answer is C. The patient is suffering from severe hypothermia. Atrial dysrhythmias are common below 32o C and are associated with a slow ventricular response. It usually converts spontaneously with rewarming. While answers B through E are all active rewarming techniques (active external - Bair Hugger, AVA rewarming, immersion, active core - peritoneal lavage), the best answer for someone with severe hypothermia with mental status change and cardiac dysrhythmias is probably active core rewarming . This technique minimizes rewarming collapse in patients with temperatures below 32o C. The patient will likely need intubated as ileus, bronchorrhea, and depressed protective airway reflexes are common with hypothermia.";"A 48 year old farmer is plowing his field when a thunderstorm rapidly overcomes him. Drivers on a nearby highway see him struck by lightening. You respond to the scene with EMS. What is the least likely finding on physical exam? A. Glascow Coma Score of 3 B. extensive skin burns C. cardiac asystole D. respiratory arrest" - correct answer "B. extensive skin burns The answer is B. A lightning strike is the discharge of a massive amount of current over a very short period of time. This often causes "short-circuiting" of electrical systems such as heart, respiratory centers, and central and autonomic nervous systems, in addition to arterial and muscular spasm. However, significant skin burns and deep tissue destruction seldom occur.";"You are treating an 80 you male in whom you suspect a lower GI bleed. Which of these following statements is TRUE regarding the guaiac test? A. A false positive may be caused by ingestion of magnesium-containing antacids B. A false positive may be caused by the presence of methylene blue C. A false negative may be caused by the presence of bromide preparations D. A false negative may be caused by the presence of chlorophyll" - correct answer "B. A false positive may be caused by the presence of methylene blue SAEM EXAM QUESTIONS 2024-2025 ACTUAL EXAM REAL EXAM 1040 QUESTIONS AND CORRECT DETAILED ANSWERS WITH RATIONALES (VERIFIED ANSWERS) The answer is B. Red fruits or meats, methylene blue, chlorophyll, iodide, cupric sulfate and bromide preparations can cause a false positive guaiac test. A false negative guaiac test can be caused by bile or ingestion of magnesium-containing antacids or ascorbic acid. Red Jell-O, tomato sauce, wine, iron therapy and Pepto-Bismol may cause the stool to look bloody when it is not.";"A 20 year old man presents to the emergency department with 1 week of intermittent bloody bowel movements associated with crampy abdominal pain, tenesmus, and fecal urgency. He is previously healthy. He is not on medications; nor has he recently traveled. What test will most likely confirm his diagnosis? A. a workup for a bleeding diathesis B. barium enema to rule out intussusception C. colonoscopy to rule out inflammatory bowel disease D. stool culture to rule out invasive bacterial diarrhea" - correct answer "C. colonoscopy to rule out inflammatory bowel disease The answer is C. This patient will need a colonoscopy with intestinal biopsy to evaluate for inflammatory bowel disease such as ulcerative colitis and Crohn's disease or other causes of colitis. Appendicitis usually presents with periumbilical pain migrating to the right lower quadrant with associated anorexia, not bloody bowel movements. Intussusception is uncommon after the age of 6. A stool culture should be obtained to rule out bacterial colitis, but his history is less suggestive of this.";"An 85 year old woman presents with acute lower abdominal pain and bloody diarrhea for 1 day. On exam her abdomen is slightly distended with diffuse tenderness. Her vital signs are stable. A plain film X-ray shows "thumbprinting" suggesting the diagnosis of: A. Volvulus B. Intussusception C. Ischemic colitis D. Invasive gastroenteritis" - correct answer "C. Ischemic colitis SAEM EXAM QUESTIONS 2024-2025 ACTUAL EXAM REAL EXAM 1040 QUESTIONS AND CORRECT DETAILED ANSWERS WITH RATIONALES (VERIFIED ANSWERS) The answer is C. Thumbprinting represents local areas of swelling in the bowel mucosa caused by submucosal edema and hemorrhage and suggests ischemic colitis.";"The most common cause of intrinsic lower gastrointestinal (GI) bleeding in an adult is: A. Cancer B. Inflammatory bowel disease C. Polyps D. Diverticulosis" - correct answer "D. Diverticulosis The answer is D. Diverticulosis and angiodysplasia account for 80% of lower GI bleeds. In approximately 10% of all patients with GI bleeding, no source of bleeding will be found.";"The most common cause of adult upper gastrointestinal (GI) bleeding is: A. Varices B. Esophagitis C. Mallory-Weiss tears D. Peptic ulcer disease (PUD)" - correct answer "D. Peptic ulcer disease (PUD) The answer is D. The most common causes of upper GI bleeding are (in descending order of frequency): PUD, gastric erosions, varices, Mallory-Weiss tears, esophagitis, and duodenitis.";"Regarding gastrointestinal bleeding, which of the following is TRUE? A. The mortality of lower gastrointestinal bleeding is higher than the mortality of upper gastrointestinal bleeding B. The most common cause of upper gastrointestinal bleeding in both adults and children is peptic ulcer disease C. Patients with a history of gastrointestinal bleeding almost always bleed again from the same site D. The majority of bleeding from diverticula occurs from the right side of the colon" - correct answer "D. The majority of bleeding from diverticula occurs from the right side of the colon